LW 7628 - Bar Prep

¡Supera tus tareas y exámenes ahora con Quizwiz!

A shareholder of a car manufacturing company wants to sue the company in federal court for breach of fiduciary duty. What information about the company, besides its place of incorporation, does the shareholder's attorney need in order to determine whether the parties are diverse? A: Where the company distributes its cars. B: Where the company has sufficient minimum contacts. C: Where the company's headquarters is located. D: Where the company's major place of manufacturing is located.

C is correct. Determining diversity for purposes of diversity jurisdiction is done with reference to the citizenship of the adverse parties. Here, because the party in question is a corporate entity, its citizenship is determined by its place of incorporation and the location of its principal place of business, which the Supreme Court has defined as a company's headquarters. Hertz Corp. v. Friend, 559 U.S. 77 (2010).

A city ordinance requires a taxicab operator's license to operate a taxicab in that city. The ordinance states that the sole criteria for the issuance of such a license are driving ability and knowledge of the geography of the city. An applicant is tested by the city for these qualifications with a detailed questionnaire, written and oral examinations, and a practical behind-the-wheel demonstration. The ordinance does not limit the number of licenses that may be issued. It does, however, allow any citizen to file an objection to the issuance of a particular license, but only on the ground that an applicant does not possess the required qualifications. City licensing officials are also authorized by the ordinance to determine, in their discretion, whether to hold an evidentiary hearing on an objection before issuing a license. A woman applies for a taxicab operator's license and is found to be fully qualified after completing the usual licensing process. Her name is then posted as a prospective licensee, subject only to the objection process. A licensed taxicab driver files an objection to the issuance of such a license to the woman solely on the ground that the grant of a license to the woman would impair the value of the licensed driver's existing license. The licensed driver demands a hearing before a license is issued to the woman so that he may have an opportunity to prove his claim. City licensing officials refuse to hold such a hearing, and they issue a license to the woman. The licensed driver petitions for review of this action by city officials in an appropriate court, alleging that the Constitution requires city licensing officials to grant his request for a hearing before issuing a license to the woman. In this case, the court should rule for A: the licensed driver, because the Due Process Clause of the Fourteenth Amendment requires all persons whose property may be adversely affected by governmental action to be given an opportunity for a hearing before such action occurs. B: the licensed driver, because the determination of whether to hold a hearing may not constitutionally be left to the discretion of the same officials whose action is being challenged. C: city officials, because the licensed driver had the benefit of the licensing ordinance and, therefore may not now question actions taken under it. D: city officials, because the licensing ordinance does not give the licensed driver any property interest in being free of competition from additional licensees.

D is correct. A person may not be deprived of liberty or property without procedural due process. The city ordinance is the source of the property right alleged here, but the scope of that property right is limited. Taxicab licenses do not include a right to be free of competition from additional licensees. Furthermore, the ordinance does not limit the number of licenses that may be issued and provides for a hearing only when the ground alleged is a lack of qualifications. Therefore, the grant of a license to the woman does not deprive the licensed driver of any property right that would require procedures to protect it. Moreover, there is no statutory right to a hearing because the only ground for a hearing is a lack of qualification, and the decision of whether to grant a pre-termination hearing is up to the discretion of the city licensing officials.

A small commercial airplane crashed in State A. The passengers and pilot, all citizens of State B, were killed in the crash. The airline that owned and operated the airplane is incorporated and has its maintenance facilities and principal place of business in State C. One day before the statute of limitations on their claims would have run, the estates of the pilot and each of the passengers filed a wrongful death action against the airline in federal court in State A. The airline was served one week later and wants to prevent the State A federal court from hearing the action. Which of the following motions is most likely to accomplish the airline's goal? A: A motion to dismiss the action for improper venue. B: A motion to dismiss the action for lack of personal jurisdiction. C: A motion to dismiss the action under the doctrine of forum non conveniens. D: A motion to transfer the action to a federal court in State C.

D is correct. When a case is originally filed in a proper venue, the court has the discretion to transfer the case if allowed under the rules governing both jurisdiction and venue. Venue was proper in State A, so a motion to transfer is the airline's only option.

A city passed an ordinance requiring individuals to obtain a license in order to care for children under the age of 12 for pay. City officials who promoted the ordinance said that the licensing process would ensure that child-care workers were adequately regulated for the health and safety of the city's children. To receive a license, the ordinance requires an individual to complete 10 hours of instruction in child care, undergo a background check, and pay a $100 fee. The ordinance affects women disproportionately to men, because female child-care workers far outnumber male child-care workers in the city. Is the ordinance constitutional? A: No, because it has a disparate impact on women without a showing that the ordinance is necessary to serve a compelling government interest. B: No, because it infringes on the freedom of contract without a compelling government interest. C: Yes, because any burden it imposes is clearly outweighed by an important government objective. D: Yes, because it is rationally related to a legitimate government objective.

D is correct. Without any evidence that the city passed the ordinance in order to disproportionately affect women, it does not amount to sex discrimination and must only satisfy mere rationality review, which it does, given its legitimate interest in protecting the health and safety of children through licensing regulation.

A woman from State A was struck by a limousine. The woman sued the limousine company, a citizen of both State B and State C, in state court in State A for $100,000 to compensate for her injuries. The limousine company timely removed the action to the local federal district court. Several months later, during discovery, the woman moved to amend her complaint to add the limousine driver, a citizen of State A, as a defendant. The woman told the court that she didn't sue the driver sooner because she did not know his name and she did not think the driver would have the resources to pay her judgment. The limousine company and the driver both argue that the woman should not be permitted to join the driver as a defendant. May the court permit the woman to join the driver as a defendant? A: Yes, because the Federal Rules have a liberal philosophy around pleading amendments. B: Yes, because courts may permit a party to amend its complaint when justice so requires. C: No, because the court would then lack subject-matter jurisdiction over the case. D: No, because parties are barred from amending their pleadings 21 days after service.

C is correct. The court may not permit the woman to amend her complaint because if she were to add the driver as a party, the joinder will destroy diversity.

Just before the statute of limitations ran on his claim, a pilot from State A sued a radiologist from State C in the federal judicial district in State A, an extremely busy court. The radiologist has never been to State A and a substantial part of the events giving rise to the claim did not occur in State A, but in State C. The radiologist moved to transfer the case to a federal court in State B because he travels to State B often for work. The pilot has family in State B and consented to the transfer. Should the court grant the motion to transfer to a federal court in State B? A: Yes, because both parties have expressed their consent to litigating the case in State B. B: Yes, because of the extremely busy docket of the federal court in State A. C: No, because State B is not a district in which the action could have properly been brought originally. D: No, because transfer is permitted only from one proper venue to another.

C is correct. The court should deny the motion because State A, the original venue, was improper. A court may only transfer from an improper venue to a venue that would have been proper originally. State B was not a venue where the case could have been properly filed originally, so the court must deny the transfer.

An actor and a limo driver are both citizens of State A. Together, they sued a talk show host, a citizen of State B, in federal district court in State B. The complaint alleged that the talk show host breached a contract in which the actor was to be a regular correspondent on the talk show and the limo driver was to provide transportation for the actor to and from the studio. The actor's alleged damages totaled $186,000 and the limo driver's alleged damages totaled $57,000. The talk show host's lawyer filed a motion to dismiss the limo driver's claim for lack of subject-matter jurisdiction. Should the court grant the talk show host's motion? A: Yes, because the limo driver and the actor are both from State A. B: Yes, because the limo driver's claim does not meet the amount in controversy requirement. C: No, because the court can exert supplemental jurisdiction over the limo driver's claim. D: No, because the limo driver consented to the personal jurisdiction by filing the lawsuit in State B.

C is correct. The court should not grant the talk show host's motion because it may assert supplemental jurisdiction over the limo driver's claim. If the federal court has diversity jurisdiction over the claim of one plaintiff, it will have supplemental jurisdiction over another plaintiff's claim IF: (i) it is so related to the first plaintiff's claim that it forms part of the same case or controversy; AND (ii) the additional plaintiff is not a citizen of the state of any of the defendants.

On January 1, 2019, a comedian from State A filed a diversity action in State A federal court against an actor from State B. The lawsuit was based on alleged wrongful business torts that occurred on January 15, 2014. The comedian is seeking $80,000 in compensatory damages. State A has a five-year statute of limitations for tort claims. The actor was served with the complaint on January 24, 2019. On January 25, 2019, the actor, upset about the lawsuit, deliberately drove his car into the comedian's car. A week later, the comedian moved to add an intentional tort claim in a supplemental pleading against the actor in the existing lawsuit. Should the court permit the comedian to supplement the pleadings with the additional intentional tort claim? A: No, because the statute of limitations for the business tort claim has already expired. B: No, because the intentional tort occurred after the comedian filed the original lawsuit. C: Yes, because the intentional tort relates back to the business tort claim, filed within the statute of limitations. D: Yes, because the intentional tort in the comedian's supplemental pleading would be a timely amendment to the original complaint.

D is correct. FRCP 15(d) allows for the comedian to supplement with the intentional tort claim because it occurred after the date of the original complaint and relates to the underlying business tort claim.

A city enacted an ordinance banning from its public sidewalks all machines dispensing publications consisting wholly of commercial advertisements. The ordinance was enacted because of a concern about the adverse aesthetic effects of litter from publications distributed on the public sidewalks and streets. However, the city continued to allow machines dispensing other types of publications on the public sidewalks. As a result of the city ordinance, 30 of the 300 sidewalk machines that were dispensing publications in the city were removed. Is this city ordinance constitutional? A: Yes, because regulations of commercial speech are subject only to the requirement that they be rationally related to a legitimate state goal, and that requirement is satisfied here. B: Yes, because the city has a compelling interest in protecting the aesthetics of its sidewalks and streets, and such a ban is necessary to vindicate this interest. C: No, because it does not constitute the least restrictive means with which to protect the aesthetics of the city's sidewalks and streets. D: No, because there is not a reasonable fit between the legitimate interest of the city in preserving the aesthetics of its sidewalks and streets and the means it chose to advance that interest.

D is correct. The city's interest in reducing litter on public sidewalks may constitute a substantial interest, however, removing machines that dispense commercial pamphlets is not a reasonable fit. Only 30 out of 300 of the machines will be removed, thus not significantly reducing the litter, and there is no indication that those 30 machines cause more litter than the remaining 270 machines.

A contractor, a citizen of State A, sued a man, a citizen of State B, in State B state court. The one-count complaint alleged breach of contract, requested damages in the amount of $79,000, and demanded a jury trial pursuant to State B rules that provide the right to jury trials in contract disputes. The man argued to the state court judge that the contractor is not entitled to a jury. Is the contractor entitled to a jury trial? A: No, because the Sixth Amendment right to a jury trial does not apply to state court breach of contract actions. B: No, because the Seventh Amendment right to a jury trial does not apply to the states. C: Yes, because the Sixth Amendment right to a jury trial applies to state court breach of contract actions. D: Yes, because State B law provides for jury trials in breach of contract actions.

D is correct. The right to a jury trial in civil cases is only guaranteed in federal courts. However, State B law provides for jury trials in contract breach cases, so the contractor is entitled to a jury trial.

A patent holder brought a patent infringement action in federal court against a licensee of the patent. The patent holder believed that a jury would be more sympathetic to his claims than a judge, and asked his lawyer to obtain a jury trial. What should the lawyer do to secure the patent holder's right to a jury trial? A: File and serve a complaint that includes a jury trial demand. B: File and serve a jury trial demand at the close of discovery. C: File and serve a jury trial demand within 30 days after the close of the pleadings. D: Make a jury trial demand at the initial pretrial conference.

A is correct. A demand for a jury trial may be included in a pleading, such as a complaint, to be secured.

A patient sued a doctor, each a private party, in an action properly brought in the federal district court of State A. Neither the patient nor the doctor filed a jury demand. Neither the patient nor the doctor served the opposing party with a jury demand within 14 days after the last pleading was served. One month after the pleadings were served, the doctor's attorney became ill and the doctor retained new counsel. The doctor's new attorney believed that the doctor would be a sympathetic figure in the eyes of a jury and filed a motion with the court requesting a jury trial. The patient opposed this motion. Is the court likely to grant the doctor's motion for a jury trial? A: Yes, because the court may order a jury trial upon a motion by a party on any issue for which a jury might have been demanded, even after the period for requesting a jury has passed. B: Yes, because a party has an absolute right to a jury trial, regardless of when the jury demand is made. C: No, because the time limit for requesting a jury trial set out in the Federal Rules of Civil Procedure bars the court from ordering a jury trial. D: No, because an untimely request for a jury trial may be granted only if all of the parties consent to a jury trial.

A is correct. A federal court MAY allow parties to request by motion a jury trial on any issue for which a jury may have been demanded. The court is not required to grant the request, but it has the authority to do so.

A plaintiff filed a federal diversity action against a corporate defendant and the same day mailed to the defendant's managing agent a notice of the complaint and two copies of a request to waive service. Sixty-one days later, after the defendant had failed to return the signed waiver request, answer, or otherwise plead, the plaintiff moved for entry of default and a default judgment. The plaintiff did not file proof of service. Assume that any relevant state service law is the same as federal law. Is the court likely to grant the motion? A: No, because the plaintiff failed to effect proper service of process. B: No, because the plaintiff failed to file proof of service. C: Yes, because the defendant should have returned the waiver request within 60 days of service. D: Yes, because the plaintiff mailed notice of the complaint as permitted by the Federal Rules of Civil Procedure.

A is correct. A plaintiff must formally serve a defendant before the defendant has an obligation to respond. Here, the defendant was not obligated and did not respond to a request for waiver of service. The plaintiff failed to effect proper service of process and an entry of default in their favor would be inappropriate.

A motorcyclist was involved in a collision with a truck. The motorcyclist sued the truck driver in state court for damage to the motorcycle. The jury returned a verdict for the truck driver, and the court entered judgment. The motorcyclist then sued the company that employed the driver and owned the truck in federal court for personal-injury damages, and the company moved to dismiss based on the state-court judgment. If the court grants the company's motion, what is the likely explanation? A: Claim preclusion (res judicata) bars the motorcyclist's action against the company. B: Issue preclusion (collateral estoppel) establishes the company's lack of negligence. C: The motorcyclist violated the doctrine of election of remedies. D: The state-court judgment is the law of the case.

A is correct. Because the motorcyclist's personal injury and property damage claims arise out of the same accident, they are part of the same cause of action and he should have brought them in one action. Because the company is in privity with the truck driver (based on the employer-employee relationship), the company cannot be found liable for the driver's acts if he is not found liable.

Current national statistics show a dramatic increase in the number of elementary and secondary school students bringing controlled substances (drugs) to school for personal use or distribution to others. In response, Congress enacted a statute requiring each state legislature to enact a state law that makes it a state crime for any person to possess, use, or distribute, within 1,000 feet of any elementary or secondary school, any controlled substance that has previously been transported in interstate commerce and that is not possessed, used, or distributed pursuant to a proper physician's prescription. This federal statute is A: unconstitutional, because Congress has no authority to require a state legislature to enact any specified legislation. B: unconstitutional, because the possession, use, or distribution, in close proximity to a school, of a controlled substance that has previously been transported in interstate commerce does not have a sufficiently close nexus to such commerce to justify its regulation by Congress. C: constitutional, because it contains a jurisdictional provision that will ensure, on a case-by-case basis, that any particular controlled substance subject to the terms of this statute will, in fact, affect interstate commerce. D: constitutional, because Congress possesses broad authority under both the General Welfare Clause and the Commerce Clause to regulate any activities affecting education that also have, in inseverable aggregates, a substantial effect on interstate commerce.

A is correct. Congress does not have the authority to require a state legislature to enact any specified legislation except when it is acting under its spending power. This is not a spending provision, so Congress cannot force states to enact these laws.

A pedestrian domiciled in State A was crossing a street in State B when he was hit by a car driven by a citizen of a foreign country. Both the pedestrian and the driver suffered injuries. The pedestrian filed a negligence action against the driver in a federal district court in State B, seeking $100,000 in damages. The driver believes that the pedestrian was crossing the street illegally and is therefore responsible for the accident. The driver seeks an attorney's advice on how best to respond to the action. Assume that State B is a contributory negligence state. How should the attorney advise the driver to respond? A: File an answer raising the affirmative defense of contributory negligence and asserting a counterclaim for negligence, seeking damages for the driver's injuries. B: File an answer raising the affirmative defense of contributory negligence and move for judgment on the pleadings. C: Move to dismiss for lack of personal jurisdiction, because the driver is not a citizen of State B. D: Move to dismiss for lack of subject-matter jurisdiction, because the driver is not a U.S. citizen.

A is correct. Contributory negligence is an affirmative defense. The defendant, in their answer, must state affirmative defenses. Counterclaims are claims the defendant may have against the plaintiff. If the counterclaim arises out of the same transaction or occurrence as the plaintiff's claim, it is a compulsory counterclaim and must be pleaded in the answer or the defendant will be barred from bringing it at a later time. In this case, contributory negligence is an affirmative defense that will defeat the plaintiff's claim, and so, should be asserted by the defendant in his answer. Additionally, the facts state that the defendant was also injured. If the defendant wants to bring a claim against the plaintiff for his injuries he must bring it now as a counterclaim, because it arises out of the same transaction or occurrence.

A corporation that was having a factory built filed a federal diversity action against the general contractor and an electrical subcontractor for the project. The complaint alleged breach of contract claims arising from construction delays and deficient electrical work that had caused the corporation significant financial losses. The general contractor filed a crossclaim against the subcontractor, asserting only the defense that the general contractor was not liable to the corporation because the subcontractor's deficient work was the sole cause of the construction delays and the corporation's losses. After discovery, the corporation settled with the general contractor and dismissed the claims against it. The subcontractor has moved to dismiss the general contractor's crossclaim. Which of the following arguments best supports the motion? A: The crossclaim is improper, because it asserts a defense, not a claim for relief. B: The crossclaim is improper, because the general contractor and the subcontractor are opposing parties, not co- parties. C: The crossclaim is invalid, because the general contractor is no longer a party to the underlying action. D: The crossclaim is premature, because it is contingent on a finding that the defendants breached the construction contract.

A is correct. Federal Rule of Civil Procedure (FRCP) 13(g) allows parties to bring claims against co-parties. However, that Rule does not allow an attempt to assert a defense against the plaintiff so as to shift direct liability for damages owed to the plaintiff to a codefendant.

A car manufacturer produced a car that was sold nationwide. Problems with the car's brakes allegedly caused several accidents and injuries. Two individual buyers of the car each filed a class action, in different states, against the manufacturer, asserting the same products liability claims on behalf of all buyers nationwide. One class action was filed in federal court and the other was filed in state court. The parties in the federal action reached a court- approved settlement, and the court entered judgment dismissing the action with prejudice. The manufacturer's attorney has moved to dismiss the state court action on the basis of res judicata (claim preclusion). Should the state court look to federal or state law to decide the effect of the judgment? A: Federal law, because the judgment was entered in federal court. B: Federal law, because the judgment was the result of a nationwide action governed by the federal class action rule. C: State law, because the judgment is being asserted in a state court. D: State law, because there is no general federal common law and preclusion is a common law doctrine.

A is correct. Federal common law governs the claim-preclusive effect of a dismissal by a federal court sitting in diversity.

For many years, a city owned and operated a reservoir that supplied the city's residents with water. Based on a finding that private entities were more efficient operators of such facilities than public entities, the city sold the reservoir to a privately owned company. The city council granted the company a non-exclusive franchise to supply water to residents of the city. After it had operated the water system for a year, the company determined that a city resident had fallen three months behind in the payment of his water bill. The company terminated the resident's water service on the basis of a company policy that provides for the summary termination of service to customers who fall more than two months behind in the payment of their bills. The policy does not provide customers an opportunity for a hearing before the termination of service. The resident sued the company in an appropriate court, asking for injunctive relief and damages. The resident claimed only that he had been deprived of property without due process of law in violation of the Fourteenth Amendment. How should the court rule? A: Dismiss the action, because the company is not a state actor for purposes of the Fourteenth Amendment. B: Dismiss the action, because the resident has not proffered payment of any of his arrearage to the company and therefore has no right to continued water service. C: Order the company to give the resident an opportunity for a hearing, because the Fourteenth Amendment applies to private entities performing public functions. D: Order the company to give the resident an opportunity for a hearing, because the Fourteenth Amendment applies to private franchises granted by public entities.

A is correct. For the action of a private entity to satisfy the "state action" requirement, the state must be "significantly involved" in the private entity. The fact that the company supplies a very important good to the public and received a non-exclusive franchise from the city is insufficient to make it a state actor. See Jackson v. Metropolitan Edison Co., 419 U.S. 345 (1974).

To encourage urban farming, a city ordinance requires vegetables sold at farmers' markets in the city to have been grown within the city limits. A farmer who grows vegetables on land in a nearby state wants to sell those vegetables at a farmers' market in the city. The farmer has challenged the ordinance on constitutional grounds. What is the farmer's best argument that the ordinance is unconstitutional? A: The ordinance discriminates on its face against interstate commerce, and there are nondiscriminatory ways to promote the city's interest in encouraging urban farming. B: The ordinance is not substantially related to an important government interest. C: The ordinance places an undue burden on interstate commerce that is clearly disproportionate to the city's interest in encouraging urban farming. D: The ordinance unconstitutionally burdens the farmer's right to travel across state lines to sell his vegetables at the farmers' market.

A is correct. On its face the ordinance discriminates against commerce from outside the city and, by doing so, it necessarily excludes commerce from outside the state. Even though the ordinance also excludes some commerce from within the state, it is still considered facially discriminatory against interstate commerce. Facially discriminatory laws are only allowed in the absence of any nondiscriminatory means to promote the government's interest. C&A Carbone, Inc. v. Town of Clarkstown, 511 U.S. 383 (1994).

A patient domiciled in State A sued a surgeon domiciled in State B in a federal court in State A, alleging claims for malpractice. The surgeon moved to dismiss the action for lack of personal jurisdiction. The court denied the motion and set discovery cutoff and trial dates. The surgeon has appealed the denial of the motion. Should the appellate court hear the merits of the surgeon's appeal? A: No, because the appellate court lacks jurisdiction over the appeal. B: No, because the district court's decision on jurisdiction is final. C: Yes, because a contrary appellate decision could terminate the action. D: Yes, because the surgeon's personal- jurisdiction challenge raises a constitutional question.

A is correct. Only final orders may be appealed to the appellate courts. Certain involuntary dismissals, such as those based on lack of jurisdiction, improper venue, or failure to join an indispensable party, do not operate as an adjudication on the merits. In this case, the motion is not appealable because it is not an adjudication on the merits, and thus the appellate court lacks jurisdiction.

A particular state has a state employee grievance system that requires any state employee who wishes to file a grievance against the state to submit that grievance for final resolution to a panel of three arbitrators chosen by the parties from a statewide board of 13 arbitrators. In any given case, the grievant and the state alternate in exercising the right of each party to eliminate five members of the board, leaving a panel of three members to decide their case. At the present time, the full board is composed of seven male arbitrators and six female arbitrators. A female state employee filed a sexual harassment grievance against her male supervisor and the state. The state's attorney exercised all of her five strikes to eliminate five of the female arbitrators. At the time she did so, the state's attorney stated that she struck the five female arbitrators solely because she believed women, as a group, would necessarily be biased in favor of another woman who was claiming sexual harassment. Counsel for the state employee eliminated four males and one female arbitrator, all solely on the grounds of specific bias or conflicts of interest. As a result, the panel was all male. When the panel ruled against the state employee on the merits of her case, she filed an action in an appropriate state court, challenging the panel selection process as a gender-based denial of equal protection of the laws. In this case, the court should hold that the panel selection process is A: unconstitutional, because the gender classification used by the state's attorney in this case does not satisfy the requirements of intermediate scrutiny. B: unconstitutional, because the gender classification used by the state's attorney in this case denies the grievant the right to a jury made up of her peers. C: constitutional, because the gender classification used by the state's attorney in this case satisfies the requirements of the strict scrutiny test. D: constitutional, because the gender classification used by the state's attorney in this case satisfies the requirements of the rational basis test.

A is correct. The state's attorney in this case intentionally excluded arbitrators based on gender. The Supreme Court has held that peremptory challenges based solely on gender are unconstitutional as a violation of the Equal Protection Clause because they reinforce negative stereotypes against women without furthering the governmental interest of a fair trial. See J.E.B. v. Alabama, 511 U.S. 127 (1994). Similarly, the state's attorney in this question is making a gender-based classification, so the state would need to show that the classification is substantially related to an important government objective. Because the state's attorney's argument was that women would simply be biased toward a female claimant, the attorney failed to demonstrate how the exclusion of female arbitrators would be substantially related to the need for a fair arbitration.

An employer, a citizen of State A, sued an employee, a citizen of State B, in State B state court. The one-count complaint sought an injunction that would prevent the employee from accepting a job that the employer claimed would have violated a valid covenant not to compete. The employee answered the complaint, in which he demanded a jury trial pursuant to the U.S. Constitution's Seventh Amendment. Is the court likely to grant the employee's demand for a jury trial? A: No, because the Seventh Amendment has not been incorporated against the states. B: No, because the Seventh Amendment only applies to suits at common law, and this is a suit in equity given that it seeks an injunction. C: Yes, because the Seventh Amendment only applies to suits at common law, and this suit involves the interpretation of a contract. D: Yes, because injunctive relief has more than de minimus value under the Seventh Amendment.

A is correct. The Seventh Amendment preserves a right to a jury trial in all suits at common law where the amount in controversy exceeds $20 in federal courts only. State law determines whether parties have a right to a jury in state courts.

An entrepreneur from State A decided to sell hot sauce to the public, labeling it "Best Hot Sauce." A company incorporated in State B and headquartered in State C sued the entrepreneur in federal court in State C. The complaint sought $50,000 in damages and alleged that the entrepreneur's use of the name "Best Hot Sauce" infringed the company's federal trademark. The entrepreneur filed an answer denying the allegations, and the parties began discovery. Six months later, the entrepreneur moved to dismiss for lack of subject-matter jurisdiction. Should the court grant the entrepreneur's motion? A: No, because the company's claim arises under federal law. B: No, because the entrepreneur waived the right to challenge subject-matter jurisdiction by not raising the issue initially by motion or in the answer. C: Yes, because although the claim arises under federal law, the amount in controversy is not satisfied. D: Yes, because although there is diversity, the amount in controversy is not satisfied.

A is correct. The claim asserts federal trademark infringement, and therefore it arises under federal law. Subject- matter jurisdiction is proper under 28 U.S.C. § 1331 as a general federal-question action. That statute requires no minimum amount in controversy, so the amount the company seeks is irrelevant.

A merchant from State A sued a banker from State B in the federal district court of State A on a state law claim that arose in State A, where most of the evidence remains. The banker moved for a change of venue to the federal district court of State B. The merchant opposed the banker's motion to change venue. There is only one federal district court located in State A and one federal district court located in State B. Under State A's choice-of-law principles, State A law will govern the claim. Had the suit been filed in State B, under State B's choice-of-law principles, State B law would have governed the claim. Should the federal district court in State A grant the banker's motion to transfer? A: No, because State A is the merchant's choice of forum, it is a proper venue, State A law applies, and most of the evidence is located in State A. B: No, because the district court of State B is an improper venue, and the merchant has not consented to it. C: Yes, because the district court of State B is a proper venue given that the defendant resides in State B. D: Yes, because venue is proper in State B, and State B law governs the claim under State B choice of law principles.

A is correct. The court should deny the banker's motion to transfer because State A is where a substantial part of the claim arose and is thus a proper venue. In this scenario, the court's decision to transfer to another proper venue is discretionary based on the convenience of the parties and witnesses and the interest of justice. Here, State A law would apply. A court sitting in State A would be more familiar with State A substantive law and most of the evidence is present in State A.

A pedestrian was struck in a crosswalk by a truck and severely injured. The pedestrian brought a federal diversity action against the driver of the truck, alleging the following in the complaint: "On January 15, 2016, on Broad Street in City A, located in State B, the defendant negligently drove a motor vehicle, striking the plaintiff. As a result, the plaintiff was physically injured, lost wages or income, suffered physical and mental pain, and incurred medical expenses of $100, 000." The truck driver has moved to dismiss the action for failure to state a claim, arguing that the complaint lacks sufficient detail. Is the court likely to grant the motion? A: No, because the complaint alleges facts showing plausible entitlement to relief. B: No, because the pedestrian has not had an opportunity to conduct discovery. C: Yes, because the complaint fails to allege facts showing probable entitlement to relief. D: Yes, because the complaint uses the term ""negligently,"" which is a legal conclusion.

A is correct. The current pleading standard requires complaints to contain sufficient facts to state a claim that is "plausible on its face." Bell Atlantic Corp. v. Twombly, 550 U.S. 544 (2007). Courts must be able to do more than infer the "possibility" of recovery. Id. Here, the pedestrian's complaint sets out the date and place of an accident in which the pedestrian claims that the truck driver hit him and lists several resulting injuries and losses. If proven, these facts set out a plausible negligence claim.

At the close of a two-week federal jury trial, the court asked the parties to submit proposed jury instructions. The court selected some of the proposed instructions, reformulated others, and charged the jury accordingly. The parties made no objections. The jury returned a verdict for the plaintiff, and the court entered judgment on the verdict. The defendant appealed, arguing that several of the court's instructions were plain error. What is the plaintiff's best argument in response? A: Any errors in the instructions did not affect the defendant's substantial rights. B: The defendant waived the appeal by failing to first move for a new trial. C: The court has discretion as to what instructions to give and can be reversed on appeal only for abuse of that discretion. D: The defendant waived the errors in the instructions by not objecting to them at trial.

A is correct. The defendant did not object to any of the instructions, so the defendant's appeal can only be successful if the instruction affected the defendant's substantial rights, which is the standard for plain error. Fed. R. Civ. P. 51(d)(2). Thus, the plaintiff's best response would be that the instructions did not affect the defendant's substantial rights.

A park ranger filed suit in state court against a scout, a rancher, and a researcher, asserting claims under federal law. The scout was properly served with the summons and complaint on February 1, 2021. The rancher was properly served with the summons and complaint on March 5, 2021. The researcher was properly served with the summons and complaint on April 7, 2021. On April 17, 2021, the researcher filed a timely notice of removal with the proper federal district court. Immediately thereafter, the scout filed his own notice of removal. Two weeks after April 17, 2021, the rancher then filed with the court a letter stating his consent to the removal of the case. The park ranger then moved for remand to state court. Should the court grant the park ranger's motion to remand? A: No, because removal was proper and the federal court has subject-matter jurisdiction over the case. B: No, because the researcher filed a timely notice of removal, which applied to all defendants. C: Yes, because the scout, rancher, and researcher were all required to simultaneously join in a single, timely, notice of removal. D: Yes, because the rancher's filing a letter of consent with the federal court to remove the case was improper.

A is correct. The defendants properly removed the case. The later-served defendant, the researcher, initiated removal, which the other defendants joined within 30 days, satisfying the consent requirement. Moreover, the federal court has subject-matter jurisdiction based on federal question.

A depositor sued her bank in federal court, seeking $30,000 in damages. The complaint set out the bank's method of calculating interest and alleged that the method violated both federal banking law and state contract law. The bank filed a motion to dismiss, asserting that its method of calculating interest, as described in the complaint, was proper under federal law and that the federal law preempted the depositor's state-law claim. The court granted the motion to dismiss the depositor's federal claim and reserved ruling on the motion to dismiss the state-law claim. After completing discovery, the bank has moved to dismiss the state-law claim for lack of subject-matter jurisdiction. How should the court proceed? A: The court may hear the state-law claim, because it is so related to the federal claim that they form part of the same case or controversy. B: The court must hear the state-law claim, because it is so related to the federal claim that they form part of the same case or controversy. C: The court must hear the state-law claim, because the bank raised a federal defense to that claim. D: The court must not hear the state-law claim, because it arises under state law and the depositor seeks less than the minimum amount in controversy.

A is correct. The dismissal of the federal claim did not divest the court of supplemental jurisdiction over the state-law claim. If a plaintiff presents a colorable claim under federal law, it will be sufficient to support supplemental jurisdiction over state-law claims that form part of the same case or controversy. The court may, in its discretion, rely on the dismissal as a reason to discontinue hearing the state-law claim but it is not required to do so.

A construction contractor brought a breach of contract claim in federal court against a homeowner who had hired the contractor to build an apartment over an existing garage. The action turned on the scope of the work covered by the contract. The contractor and the homeowner were the only witnesses at the bench trial, and they strongly disagreed about the scope of the work. At the end of the trial, the judge stated findings of fact on the record but never issued a written opinion. Neither party objected to the findings. The judge found in favor of the homeowner, and the contractor appealed. Is the appellate court likely to overturn the findings? A: No, because the appellate court must give due regard to the trial judge's opportunity to determine witness credibility. B: No, because the contractor failed to object to the findings when the judge stated them in open court. C: Yes, because a judge must set forth findings of fact in a written opinion or memorandum of decision. D: Yes, because there were disputed issues of fact at trial.

A is correct. The judge made findings of fact based on the witnesses' testimony. The appellate court will not overturn these findings unless clearly erroneous, a very high standard to meet.

A patent holder brought a patent infringement action in federal court against a licensee of the patent and the case went to a jury trial. At trial, the patent holder failed to object to a proposed jury instruction both before the court instructed the jury and before the jury retired to consider the verdict. During jury deliberations, however, the patent holder objected to the jury instructions as being erroneous. The jury then returned a verdict for the licensee. The patent holder filed an appeal claiming the trial court erred in not modifying the jury instruction following his objection. Is the appellate court likely to uphold the trial court's refusal to modify the jury instruction? A: Yes, because the patent holder waived the objection by not timely raising it before the jury began deliberating. B: Yes, because the Federal Rules require objections to jury instructions to be made before the jury is instructed and before arguments are delivered. C: No, because the patent holder objected to the jury instructions while the jury was still deliberating. D: No, because there are no time limits on when objections to jury instructions may be made.

A is correct. The patent holder waived the objection to the instructions because of the failure to object at the first opportunity to do so (or at all), which is required to properly preserve such a claim on appeal.

In order to ensure the high quality of tutors in state public schools, a state law requires public schools to hire only tutors licensed by the state. To obtain a license, a tutor is required to achieve a passing score on a state-administered exam in the tutor's area of specialty. On several occasions, an organization representing tutors licensed in the state has successfully lobbied against proposed legislation that would have eased the licensing requirement. An out-of-state tutoring company would like to conduct business in the state, but very few of its tutors are licensed by the state. The company has sued to challenge the law. Is the company likely to prevail? A: No, because the law is rationally related to a legitimate state interest. B: No, because the law is substantially related to an important state interest. C: Yes, because the law serves the special interests of licensed tutors rather than a legitimate government interest. D: Yes, because the law violates the privileges and immunities clause of Article IV.

A is correct. The state licensing requirement should receive rational basis scrutiny because it neither burdens a fundamental individual right nor creates a constitutionally suspect classification. The licensing requirement is a rational means of protecting the state's legitimate interest in ensuring the competency of tutors. Therefore, the state law satisfies rational basis review and is constitutional.

A student was riding on a bus owned and operated by a travel company when the bus collided with a truck in State B, thereby injuring the student. The student then brought a jurisdictionally valid lawsuit against the travel company in state court in State A. The court found the travel company not negligent and dismissed the student's lawsuit. A teacher had been riding on the same bus as the student and thereafter brought a jurisdictionally valid lawsuit against the travel company in State B federal court for injuries. Will the travel company's finding of non-liability in the student's case apply in the teacher's lawsuit? A: No, because the teacher was not a party in the first lawsuit. B: No, because non-mutual collateral estoppel is unconstitutional. C: Yes, because due process permits the use of non-mutual collateral estoppel against a non-party to the first lawsuit. D: Yes, because a finding from State A state court is binding on a State B federal court.

A is correct. The teacher was not a party in the first lawsuit, so the travel company may not use defensive collateral estoppel to prevent any of the same issues that were tried in the first case from being relitigated in the teacher's lawsuit.

The open-air amphitheater in the city park of a town has been utilized for concerts and other entertainment programs. Until this year, each of the groups performing in that city facility was allowed to make its own arrangements for sound equipment and sound technicians. After recurring complaints from occupants of residential buildings adjacent to the city park about intrusive noise from some performances held in the amphitheater, the town's city council passed an ordinance establishing city control over all sound amplification at all programs held there. The ordinance provided that the town's Department of Parks would be the sole provider in the amphitheater of sound amplification equipment and of the technicians to operate the equipment "to ensure a proper balance between the quality of the sound at such performances and respect for the privacy of nearby residential neighbors." Which of the following standards should a court use to determine the constitutionality on its face of this content neutral ordinance? A: The ordinance is narrowly tailored to serve a substantial government interest, and does not unreasonably limit alternative avenues of expression. B: The ordinance is rationally related to a legitimate government interest, and does not unreasonably limit alternative avenues of expression. C: The ordinance is rationally related to a legitimate government interest and does not restrict the expressive rights involved any more than is reasonable under the circumstances. D: The ordinance is substantially related to a legitimate governmental interest and does not restrict the expressive rights involved any more than is reasonable in light of the surrounding circumstances.

A is correct. This answer choice states the correct standard: the ordinance must be narrowly tailored to serve a substantial government interest and does not unreasonably limit alternative avenues of expression.

The constitution of a state authorizes a five-member state reapportionment board to redraw state legislative districts every ten years. In the last state legislative reapportionment, the board, by a unanimous vote, divided the greater metropolitan area, composed of a large city and several contiguous townships, into three equally populated state legislative districts. The result of that districting was that 40% of the area's total Black population resided in one of those districts, 45% of the area's total Black population resided in the second of those districts, and 15% resided in the third district. A registered voter, who is Black and is a resident of the city, brings suit in an appropriate court against the members of the state reapportionment board, seeking declaratory and injunctive relief that would require the boundary lines of the state legislative districts in the greater metropolitan area be redrawn. His only claim is that the current reapportionment violates the Fifteenth Amendment because it improperly dilutes the voting power of the Black residents who reside in that area. If no federal statute is applicable, which of the following facts, if proven, would most strongly support the validity of the action of the state reapportionment board? A: In drawing the current district lines, the reapportionment board precisely complied with state constitutional requirements that state legislative districts be compact and follow political subdivision boundaries to the maximum extent feasible. B: The reapportionment board was composed of three white members and two Black members and both of the board's Black members were satisfied that its plan did not improperly dilute the voting power of the Black residents who reside in that area. C: Although the rate of voter registration among Blacks is below that of voter registrations among whites in the greater metropolitan area, two Black legislators have been elected from that area during the last 15 years. D: The total Black population of the greater metropolitan area amounts to only 15% of the population that is required to comprise a single legislative district.

A is correct. This offers the strongest support for the validity of the action because it provides an alternate reasoning for the redistricting, one that does not involve race as a predominant factor, and would therefore not trigger strict scrutiny.

A passenger domiciled in State A has brought a federal diversity action in State A against a railroad, seeking damages for injuries suffered when the railroad's train, which the passenger boarded in State A, derailed in State B. The railroad is incorporated and has its principal place of business in State B and operates in States A and B. Several other passengers were also injured and have brought individual actions in State B federal court. The railroad has moved the State A federal court to transfer the action to the State B federal district court. State B has only one federal district. What is the railroad's best argument in support of the motion? A: The accident occurred in State B and many witnesses are located there. B: The action involves common questions of fact with similar actions pending in State B. C: The railroad is incorporated and has its principal place of business in State B. D: Venue is not proper in State A.

A is correct. Transfers to a different federal court are granted in the interest of justice and for the convenience of the parties and witnesses. Here, because the accident occurred in State B, the railroad is located in State B, and other passengers have brought actions in State B federal court, State B would be the most convenient venue for the action.

A state law provides that a person who has been divorced may not marry again unless he or she is current on all child-support payments. A woman who was refused a marriage license pursuant to this law sued the appropriate state officials. What standard should the court apply in reviewing the constitutionality of this law? A: The state must show that the law is necessary to serve a compelling government interest. B: The state must show that the law is substantially related to an important government interest. C: The woman must show that the law serves no important public purpose. D: The woman must show that the legislature did not have a rational basis for enacting the law.

A is correct. U.S. Supreme Court precedent establishes that an individual's decision to marry is a fundamental right, and therefore, laws that unduly burden a decision to marry trigger strict judicial scrutiny. This strict scrutiny standard obligates the state to prove that the law is necessary to serve a compelling government interest. Here, the law burdens the right to marry by requiring that a person who has been divorced may not remarry unless he or she is current on all child support payments. Because this law burdens the fundamental right to marry, the court will apply the strict scrutiny standard and require the state to prove that the law is necessary to serve a compelling state interest.

A plaintiff domiciled in State A sued a defendant domiciled in State B, alleging that the defendant had failed to deliver products to the plaintiff under an ongoing contract. The plaintiff attached to the complaint an invoice for the products marked "paid" and a receipt for a wire transfer to the defendant in the amount of $80,000. The defendant answered the complaint, denying that the plaintiff had suffered any losses and asserting a counterclaim for nonpayment of $90,000 for an earlier delivery of products. In response to the counterclaim, the plaintiff has moved for judgment on the pleadings. How is the court likely to rule? A: Deny the motion as premature, because the plaintiff has not yet answered the counterclaim. B: Deny the motion, because a plaintiff that attaches items to the complaint cannot move for judgment on the pleadings. C: Grant the motion, because the defendant failed to cross-move. D: Grant the motion, because the defendant has not denied that it failed to deliver products after payment by the plaintiff.

A is correct. Under Federal Rule of Civil Procedure (FRCP) 12(c), a judgment on the pleadings may be made after the pleadings are closed. However, because the defendant in this case asserted a counterclaim in its answer, the plaintiff must file an answer to the counterclaim under FRCP 7(a)(3) before making any motions.

A woman sued her former employer in state court, asserting age and sex discrimination claims under both state and federal law. The woman's attorney had recently been embarrassed in court by the judge to whom the case was assigned. Wishing to avoid difficulties with the judge, the woman's attorney promptly removed the case to federal court on the basis of federal-question jurisdiction. The employer's attorney has timely moved to remand. How is the federal court likely to proceed? A: Remand the entire case. B: Remand the state claims but keep the federal claims. C: Retain the case to avoid the risk of bias and impropriety in having it proceed before a judge who has shown clear hostility toward the woman's attorney. D: Retain the case, because it was timely removed and the woman alleges federal claims.

A is the correct answer. A defendant can remove an action that could have originally been brought by the plaintiff in the federal courts. Only defendants can exercise the right of removal. In this case, the woman is the plaintiff. Therefore, her attorney may not file for removal. The case should be remanded for a defect in the removal.

A worker from State A sued a farmer from State B in a state court in State B, alleging that the farmer violated the worker's rights under Title VII of the Civil Rights Act of 1964. The farmer filed a notice of removal in the federal court of State B. The worker then filed a motion for remand to state court. Should the court grant the worker's motion to remand? A: No, because the federal claim was mistakenly filed in a state court. B: No, because the court has subject-matter jurisdiction based on a federal question. C: Yes, because the notice of removal should have been filed with the state court. D: Yes, because the farmer is a citizen of State B, the state where the action was brought.

B is correct. Because the complaint alleges a violation of Title VII of the Civil Rights Act of 1964, a federal law, the farmer can remove to federal court in State B.

A clerical worker has been employed for the past two years in a permanent position in the County Public Records Office in a particular state. The clerk has been responsible for copying and filing records of real estate transactions in that office. The clerk works in a nonpublic part of the office and has no contact with members of the public. However, state law provides that all real estate records in that office are to be made available for public inspection. On the day an attempted assassination of the governor of the state was reported on the radio, the clerk remarked to a coworker, "Our governor is such an evil man, I am sorry they did not get him." The clerk's coworker reported this remark to the clerk's employer, the county recorder. After the clerk admitted to making the remark, the county recorder dismissed him stating that "there is no room in this office for a person who hates the governor so much." The clerk sued for reinstatement and back pay. His only claim is that the dismissal violated his constitutional rights. In this case, the court should hold that the county recorder's dismissal of the clerk was A: unconstitutional, because it constitutes a taking without just compensation of the clerk's property interest in his permanent position with the county. B: unconstitutional, because in light of the clerk's particular employment duties his right to express himself on a matter of public concern outweighed any legitimate interest the state might have had in discharging him. C: constitutional, because the compelling interest of the state in having loyal and supportive employees outweighs the interest of any state employee in his or her job or in free speech on a matter of public concern. D: nonjusticiable, because public employment is a privilege rather than a right and, therefore, the clerk lacked standing to bring this suit.

B is correct. Because the employee's comments had no impact on the performance of his official duties, and given that he is a citizen with the right to express his views, the court should hold that his termination was unconstitutional.

A pedestrian from State A sued a motorcyclist from State B. The pedestrian properly filed suit in a state court located in the Eastern District of State A and sought $100,000 in damages for the tortious injuries caused by the motorcyclist's allegedly negligent acts in the Western District of State A. The motorcyclist filed a notice of removal in the federal court for the Eastern District of State A, which geographically embraces the location of where the action was originally filed. Is removal proper? A: Yes, because the requirements of complete diversity are satisfied. B: Yes, because the lawsuit was filed in state court in the Eastern District of State A. C: No, because the motorcyclist is a citizen of State B. D: No, because the accident occurred in the Western District of State A.

B is correct. Because the suit was originally filed in the state court that geographically sits in the Eastern District of State A, the federal court in the Eastern District of State A is a proper venue for removal.

After being fired, a woman sued her former employer in federal court, alleging that her supervisor had discriminated against her on the basis of her sex. The woman's complaint included a lengthy description of what the supervisor had said and done over the years, quoting his telephone calls and emails to her and her own emails to the supervisor's manager asking for help. The employer moved for summary judgment, alleging that the woman was a pathological liar who had filed the action and included fictitious documents in revenge for having been fired. Because the woman's attorney was at a lengthy out-of-state trial when the summary-judgment motion was filed, he failed to respond to it. The court therefore granted the motion in a one-line order and entered final judgment. The woman has appealed. Is the appellate court likely to uphold the trial court's ruling? A: No, because the complaint's allegations were detailed and specific. B: No, because the employer moved for summary judgment on the basis that the woman was not credible, creating a factual dispute. C: Yes, because the woman's failure to respond to the summary-judgment motion means that there was no sworn affidavit to support her allegations and supporting documents. D: Yes, because the woman's failure to respond to the summary-judgment motion was a default giving sufficient basis to grant the motion.

B is correct. By challenging the woman's credibility in its motion, the employer disputed all the facts and evidence she had laid out in her complaint. Therefore, the motion did not meet the standard for summary judgment, and the trial court should be reversed.

A day before the applicable statutory limitations period expired, a worker filed a federal diversity action for defamation against her former employer, alleging that the employer had falsely and publicly accused her of stealing trade secrets. In describing the events that led to the false accusations, the complaint quoted a statement of a competitor made to the employer about the worker's alleged theft. During discovery, the worker deposed the competitor. One week after discovery closed, the worker moved to amend the complaint to add the competitor as a defendant. The competitor opposed the motion on the ground that the statutory limitations period had expired. Is the court likely to grant the motion? A: No, because discovery has closed, and the competitor will be prejudiced. B: No, because the amendment would not relate back and thus would be futile. C: Yes, because leave to amend should be freely granted when the underlying action was timely. D: Yes, because the allegations against the competitor arise out of the same factual circumstances and relate back to the allegations in the original complaint.

B is correct. Courts may deny leave to amend when the claim to be added is futile. Here, the claim is futile on limitations grounds unless relation back applies. Under Federal Rule of Civil Procedure (FRCP) 15, relation back of amendments that add a party will be permitted if the failure to name the party is the result of a mistake in identity. The facts provide that the worker's failure to name the competitor in the original complaint was not due to a mistake of identity, but was instead a strategic choice. Thus, the amendment does not relate back and, since the claim against the competitor comes after the applicable limitations period expired, the claim is futile.

To express their disagreement with a city ordinance prohibiting public nudity, a group of demonstrators performed a dance in the nude in a city park. The demonstrators were convicted of violating the ordinance. Do the convictions violate the First Amendment? A: No, because the demonstrators' conduct is not protected by the First Amendment. B: No, because the demonstrators' protected expression may be validly subject to content-neutral regulation such as the public nudity ordinance. C: Yes, because the city park is a public forum. D: Yes, because the demonstrators' conduct constitutes protected speech and the ordinance is not narrowly tailored to further a substantial government interest.

B is correct. Nude dancing is expressive conduct protected by the First Amendment, but the Supreme Court has nevertheless upheld bans on nude dancing and public nudity more generally. The ordinance here is a content-neutral prohibition because it does not target any message conveyed by public nudity. Therefore, based on these grounds, the convictions do not violate the First Amendment.

Although the U.S. Supreme Court has held that an out-of-state citizen's deliberate sale of a single item in another state may give rise to personal jurisdiction, State B's highest court has held that such a sale is not sufficient for purposes of State B's long-arm statute. A novelist from State C sold an expensive machine to a mathematician from State B. After the sale, the mathematician sued the novelist in the federal district court in State B, alleging the novelist's breach of contract and seeking $125,000 in damages. The novelist's only contact with State B is the sale of that machine. May the federal district court in State B hear the case? A: No, because venue is improper, even though the court has personal jurisdiction over the novelist. B: No, because the court does not have personal jurisdiction over the novelist. C: Yes, because the court has personal jurisdiction over the novelist and venue is proper. D: Yes, because the court has subject-matter jurisdiction over the lawsuit.

B is correct. State B's long-arm statute has been interpreted to preclude a single sale in the state from establishing personal jurisdiction over a defendant. Although the facts support subject-matter jurisdiction (based on diversity) and venue (the sale occurred in State B), without personal jurisdiction over the novelist, the court may not hear the case.

In an effort to counteract a steep increase in juvenile crime, a state enacted a law terminating the parental rights of any state resident whose child under 16 years of age is convicted of a violent crime in the state. The law directs the state juvenile court to enter a termination order in such a case after the parent has been afforded notice and an opportunity for a hearing at which the only relevant issues are the age of the child and whether the child has been convicted of a violent crime in the state. Is the state law constitutional? A: No, because the law is not narrowly tailored to serve a substantial state interest. B: No, because the law is not necessary to serve a compelling state interest. C: Yes, because a state's police power authorizes it to punish criminal behavior with appropriate sanctions. D: Yes, because the law is rationally related to a legitimate state interest.

B is correct. The Court uses the strict scrutiny standard when a suspect classification or fundamental right is involved. Under this standard, a law will be upheld only if it is necessary to achieve a compelling or overriding government purpose. The custody of one's children has been held to be a fundamental right, and therefore strict scrutiny will apply. Here, there is a state interest in decreasing juvenile crime. However, there are less burdensome ways of decreasing crime.

Congressional hearings determined that the use of mechanical power hammers is very dangerous to the persons using them and to persons in the vicinity of the persons using them. As a result, Congress enacted a statute prohibiting the use of mechanical power hammers on all construction projects in the United States. Subsequently, a study conducted by a private research firm concluded that nails driven by mechanical power hammers have longer- lasting joining power than hand-driven nails. After learning about this study, a city council enacted an amendment to its building safety code requiring the use of mechanical power hammers in the construction of all buildings intended for human habitation. This amendment to the city's building safety code is A: unconstitutional, because it was enacted subsequent to the federal statute. B: unconstitutional, because it conflicts with the provisions of the federal statute. C: constitutional, because the federal statute does not expressly indicate that it supersedes inconsistent state or local laws. D: constitutional, because the long-term safety of human habitation justifies some additional risk to the people engaged in their construction.

B is correct. The Supremacy Clause of the U.S. Constitution provides that a valid federal law controls when a state or local law conflicts with it. The law here is a valid exercise of Congress's commerce powers, and because the local law conflicts with it, the local law is unconstitutional.

An investor from State A sued a State A company in State A court, asserting two claims: (i) a violation of federal securities laws, over which state and federal courts have concurrent subject-matter jurisdiction; and (ii) fraud under State A law. The company filed a notice of removal in the local federal district court. Several months later, after seeking the advice of counsel, the investor decided to voluntarily dismiss his federal claim and moved to remand the case to state court. May the court grant the investor's motion to remand? A: Yes, because if the federal court lacks subject-matter jurisdiction at any time before final judgment, the case must be remanded to state court. B: Yes, because the federal court has dismissed all claims over which there existed an independent basis of jurisdiction and may use its discretion to remand. C: No, because the federal court still has subject-matter jurisdiction, even after dismissal of the federal claim, and is therefore obligated to hear the case. D: No, because the motion to remand was made more than 30 days after the filing of the notice of removal and is therefore untimely.

B is correct. The court initially exerted supplemental jurisdiction over the investor's state law claim under federal question jurisdiction, and the dismissal of the federal claim allows the court to exercise its discretion to decline to hear the remaining claim without an independent basis for jurisdiction.

A personal assistant from State B sued an actor from State A in federal district court in State B, alleging a breach of contract. The alleged breach occurred while the personal assistant was working for the actor on a one-day shoot in State B. Aside from the one-day shoot, the actor has had no other contacts with State B. The personal assistant had worked for the actor for over five years, with all prior services performed in State A. The personal assistant is seeking $125,000 in damages. In response to the complaint, the actor filed a motion to dismiss for lack of personal jurisdiction. Should the court grant the actor's motion to dismiss? A: No, because the actor's challenge to personal jurisdiction was improperly asserted in a motion to dismiss. B: No, because the actor's presence in State B for the one-day shoot was sufficient to establish minimum contacts with State B. C: Yes, because the actor did not have sufficient minimum contacts with State B to establish personal jurisdiction. D: Yes, because personal jurisdiction may not be waived, even though the motion to dismiss was improperly filed.

B is correct. The court should not grant the actor's motion to dismiss because, by contracting with the personal assistant for services performed in State B, even if only for one day, the actor purposefully availed himself of the forum. This is despite the fact that most of the services performed between the parties occurred in State A, where the actor resides. By hiring the personal assistant and receiving services in State B, the actor should have foreseen that he could be subject to litigation in that forum. For a federal court to properly exercise personal jurisdiction over a party, that party must have had "minimum contacts" with the forum. This is often defined as taking actions that purposefully avail himself of the privilege of doing business within the state. A key determination here is whether it was foreseeable, based on the party's actions, that he may be subject to litigation in the state.

A shop owner domiciled in State A sued a distributor in a federal district court in State A for breach of a contract. The shop owner sought $100,000 in damages for allegedly defective goods that the distributor had provided under the contract. The distributor is incorporated in State B, with its principal place of business in State C. The distributor brought in as a third-party defendant the wholesaler that had provided the goods to the distributor, alleging that the wholesaler had a duty to indemnify the distributor for any damages recovered by the shop owner. The wholesaler is incorporated in State B, with its principal place of business in State A. The wholesaler has asserted a $60,000 counterclaim against the distributor for payment for the goods at issue, and the distributor has moved to dismiss the counterclaim for lack of subject-matter jurisdiction. Should the motion to dismiss be granted? A: No, because the wholesaler's and the distributor's principal places of business are diverse. B: No, because there is supplemental jurisdiction over the wholesaler's counterclaim. C: Yes, because there is no diversity of citizenship between the distributor and the wholesaler. D: Yes, because there is no diversity of citizenship between the shop owner and the wholesaler.

B is correct. The distributor and the wholesaler are not diverse and the amount in controversy does not exceed $75,000. However, the claim can still be added through supplemental jurisdiction because the claim arises from the same common nucleus of operative fact as the underlying lawsuit, thus establishing subject-matter jurisdiction.

A woman met with a heart surgeon and his nurse practitioner about having surgery to fix a blockage in her artery. The heart surgeon and nurse together described the procedure and informed her of the risks. The woman agreed to have the surgery exactly one week later and signed the consent form. One week later, the surgeon performed the surgery on the woman and the nurse practitioner was not present. The surgery went badly. On the last day before the statute of limitations for tort claims in that state ran out, unhappy with the results, the woman filed a single-count complaint against the heart surgeon, as the sole defendant, in federal court. The complaint's only assertion was that the heart surgeon failed to obtain informed consent because he did not disclose an alternative method he could have used to perform the surgery. Four weeks after the initial complaint, the woman sought to amend the complaint to add a negligence claim against the heart surgeon on the theory that he poorly performed the surgery and post-operative procedures. Is the court likely to allow the woman to amend her complaint? A: No, because the statute of limitations expired the day after the woman filed her initial complaint and she did not add the related negligence claim until four weeks later. B: No, because the negligence claim around the physician's performance during and after the surgery did not arise out of the same event set forth in the original complaint. C: Yes, because the heart surgeon knew or should have known that the woman would have also brought a negligence claim against him, but for a mistake. D: Yes, because the negligence claim relates back to the date of the original complaint, which was filed within the statute of limitations.

B is correct. The event or transaction set forth in the original complaint was the physician and nurse's advising about the options for surgery, which the woman later claimed did not amount to informed consent. By contrast, the negligence claim is about the physician's performance during and after surgery, where the nurse was not present. The claims are not based on the same events, so the court is unlikely to allow the woman to amend with the negligence claim.

A landscaper from State A sued a homeowner from State B in a state court in State B. The landscaper is seeking $100,000 as compensation for tortious injuries caused by the homeowner's allegedly negligent acts in State B. The homeowner filed a notice of removal in the federal court in State B. The landscaper filed a motion to remand to the state court. Should the court grant the landscaper's motion to remand? A: No, because the federal court may use supplemental jurisdiction to establish subject-matter jurisdiction. B: No, because the federal court has subject-matter jurisdiction under the diversity statute. C: Yes, because the homeowner is a citizen of State B, the state in which the action was brought. D: Yes, because the notice of removal should have been filed in state court.

C is correct. As a citizen of the state where the original action was brought, the homeowner's removal to federal court was improper and the court should grant the landscaper's motion to remand.

State X's rules prohibit personal injury plaintiffs from alleging the damages amount in their complaints. A janitor from State X sued a mechanic from State A for negligence in a State X court. In his complaint, the janitor alleged that, due to the mechanic's negligence, he suffered multiple broken bones that required extensive surgeries and sustained brain damage. The janitor sought compensation for those injuries for pain and suffering and reimbursement of medical expenses. The parties participated in discovery for over a year, at which point the mechanic received timely answers to interrogatories that, for the first time, expressly indicated that the janitor sought $1 million in damages. The mechanic promptly filed a notice of removal with the federal district court, citing that the janitor's interrogatory answer that referenced $1 million in damages. The janitor moved to remand the action. Should the court grant the janitor's motion to remand? A: Yes, because the mechanic failed to prove by a preponderance of the evidence that the amount in controversy exceeds $75,000. B: Yes, because the mechanic's removal to federal court occurred more than one year after the janitor filed the claim. C: No, because the janitor deliberately failed to disclose the amount in controversy to prevent the mechanic from removing the case to federal court. D: No, because the mechanic promptly removed the case after ascertaining from the interrogatory answers that the janitor was seeking $1 million in damages.

B is correct. The mechanic had up to one year to remove this diversity action to federal court, but he waited until over a year had passed, which means it was untimely and the court should remand.

A private university is owned and operated by a religious organization. The university is accredited by the department of education of the state in which it is located. This accreditation certifies that the university meets prescribed educational standards. Because it is accredited, the university qualifies for state funding for certain of its operating expenses. Under this funding program, 25 percent of the university's total operating budget comes from state funds. A professor at the university was a part-time columnist for the local newspaper. In one of her published columns, the professor argued that "religion has become a negative force in society." The university subsequently discharged the professor, giving as its sole reason for the dismissal her authorship and publication of this column. The professor sued the university, claiming only that her discharge violated her constitutional right to freedom of speech. The university moved to dismiss the professor's lawsuit on the ground that the U.S. Constitution does not provide the professor with a cause of action in this case. Should the court grant the university's motion to dismiss? A: Yes, because the First and Fourteenth Amendments protect the right of the university to employ only individuals who share and communicate its views. B: Yes, because the action of the university in discharging the professor is not attributable to the state for purposes of the Fourteenth Amendment. C: No, because the accreditation and partial funding of the university by the state are sufficient to justify the conclusion that the state was an active participant in the discharge of the professor. D: No, because the U.S. Constitution provides a cause of action against any state-accredited institution that restricts freedom of speech as a condition of employment.

B is correct. The professor's discharge by the private university did not constitute "state action," which is required to trigger Fourteenth Amendment protections.

A buyer filed suit against a seller in federal court in State A. The federal district court entered a default judgment in favor of the buyer. Within one month after the entry of the default judgment, the seller made a motion seeking relief from the judgment on the ground of excusable neglect. The seller made a prima facie showing that it had a meritorious defense to the buyer's claim and that setting aside the default judgment would not unfairly prejudice the buyer. However, the record also showed that the seller had been served with the summons and complaint and knew of the action in time to appear, plead, or otherwise defend against it, but failed to do any of those things. Is the court likely to grant the seller's motion seeking relief from judgment? A: No, because the court may simply set aside the entry of default judgment against the seller. B: No, because the seller's failure to appear, plead, or otherwise defend against the lawsuit does not constitute a form of excusable neglect. C: Yes, because of the policy that courts will liberally grant relief from default judgments. D: Yes, because the seller met the burden for relief from a default judgment and the buyer would not be unfairly prejudiced by having the default judgment set aside.

B is correct. The seller failed to take any action in response to the buyer's claim despite the seller's awareness of the suit filed against him. Such behavior provides no "excuse" and thus does not justify a court relieving the seller from the final judgment entered against him.

A city has had a severe traffic problem on its streets. As a result, it enacted an ordinance prohibiting all sales to the public of food or other items by persons selling directly from trucks, cars, or other vehicles located on city streets. The ordinance included an inseverable grandfather provision exempting from its prohibition vendors who, for 20 years or more, have continuously sold food or other items from such vehicles located on the streets of the city. A retail vendor of ice cream products qualifies for this exemption and is the only food vendor that does. A yogurt retailer has a business similar to the ice cream vendor, but the yogurt vendor has been selling to the public directly from trucks located on the streets of the city only for the past ten years. The yogurt vendor filed suit in an appropriate federal district court to enjoin the enforcement of this ordinance on the ground that it denies the yogurt vendor the equal protection of the laws. In this case, the court will probably rule that the ordinance is A: constitutional, because it is narrowly tailored to implement the city's compelling interest in reducing traffic congestion and, therefore, satisfies the strict scrutiny test applicable to such cases. B: constitutional, because its validity is governed by the rational basis test, and the courts consistently defer to economic choices embodied in such legislation if they are even plausibly justifiable. C: unconstitutional, because the nexus between the legitimate purpose of the ordinance and the conduct it prohibits is so tenuous and its provisions are so underinclusive that the ordinance fails to satisfy the substantial relationship test applicable to such cases. D: unconstitutional, because economic benefits or burdens imposed by legislatures on the basis of grandfather provisions have consistently been declared invalid by courts as per se violations of the Equal Protection Clause of the Fourteenth Amendment.

B is correct. This ordinance regulates who may sell food from vehicles on the streets, which implicates neither a fundamental right nor a suspect class. Therefore, the ordinance would be subject to rational basis review, and the court will defer to any economic choice made by the city as long as it is plausibly justifiable.

A plaintiff, a citizen of State A, sued a defendant, a citizen of State B, in a state court in State B. The complaint asserted a $120,000 claim under state law. Fifty days after service of the complaint, the plaintiff amended the complaint, adding a second defendant, a citizen of State C. The amended complaint asserted the $120,000 state-law claim and a related federal-law claim against both defendants. Twenty-five days after service of the amended complaint, the second defendant removed the action to a federal court in State B with the first defendant's consent. Was removal proper? A: No, because the first defendant was a citizen of the forum state. B: No, because the notice of removal was untimely. C: Yes, because the amended complaint asserted a claim under federal law. D: Yes, because the parties met the requirements for diversity jurisdiction.

C is correct. An action is removable if it could originally have been filed in federal court. Here, the amended complaint contained a claim that qualified for federal-question jurisdiction. Although there was also diversity jurisdiction, removal was not barred by the forum-defendant rule, which provides that a suit that is removable solely on the basis of diversity may not be removed if any of the parties in interest, properly joined and served, is a citizen of the state in which such action is brought. That rule bars removal only when diversity is the sole basis for federal jurisdiction, unlike in this case.

Before filing a federal civil action against a seller, a buyer's attorney unsuccessfully tried to settle with the seller's attorney. Three days before the limitations period on the buyer's claim expired, the buyer's attorney told the seller's attorney that she would file a complaint that day and asked the seller's attorney whether he would accept service of the summons and complaint. The seller's attorney agreed to do so. The buyer's attorney promptly filed the complaint but forgot to serve the seller's attorney. Four months later, the buyer's attorney received a voicemail from the seller's attorney asking whether she had ever filed the buyer's complaint. The buyer's attorney immediately mailed a copy of the complaint to the seller's attorney. The following week, the seller's attorney moved to dismiss the complaint for failure to effect timely service of process. Is the court likely to grant the seller's motion to dismiss? A: No, because under the Federal Rules of Civil Procedure, the filing of the complaint commences an action and the buyer's complaint was timely filed. B: No, because the seller's attorney had notice of the complaint and agreed to accept service. C: Yes, because the buyer's attorney did not show good cause for her failure to effect timely service. D: Yes, because the limitations period expired without timely service.

C is correct. Federal Rule of Civil Procedure (FRCP) 4(m) requires that service be made within 90 days after the complaint is filed with the court. Because the buyer's attorney did not effect timely service, the only way to survive the seller's motion to dismiss is for the buyer to demonstrate good cause for failing to do so. Fed. R. Civ. P. 4(m). "Good cause" is typically something beyond the party's control. Forgetting to effect service does not constitute good cause.

A company incorporated and headquartered in State A brought a federal diversity action in State A against an attorney domiciled in State B. The complaint asserted that the attorney had sent defamatory emails to the company's customers in many different states, including State A. The attorney had few contacts with State A, and none of the contacts in that state were related to the alleged defamatory emails. After being served in State B with the summons and complaint, the attorney determined that he was not subject to personal jurisdiction in State A and ignored the action. On the company's motion, the clerk entered a default in the action. After a hearing on damages, the court entered a default judgment for the company. The attorney has moved for relief from the default judgment. What is the attorney's best argument in support of the motion? A: The attorney has a meritorious defense to the action, which he should be allowed to assert. B: The attorney's failure to respond to the service of process was reasonable under the circumstances. C: The default judgment is void, because the court did not have personal jurisdiction over the attorney. D: The default judgment was entered as a result of the attorney's mistaken belief that he need not respond.

C is correct. Federal Rule of Civil Procedure (FRCP) 60(b)(4) specifically authorizes relief if the "judgment is void." Here, the court had no personal jurisdiction over the attorney in the action, so the judgment is void, and the court may relieve the attorney from the judgment.

An illustrator from State B sued his former employer, a State A corporation, in a federal district court in State B. The illustrator had worked at the corporation's headquarters in State B. The complaint asserted that the corporation: (i) violated federal employment law; and (ii) breached the illustrator's employment contract. The illustrator is seeking $50,000 in damages. The corporation filed a motion to dismiss, asserting that the court lacks subject-matter jurisdiction over the illustrator's employment contract claim. Should the court grant the corporation's motion to dismiss? A: Yes, because the corporation is incorporated in State A, which means there is no complete diversity. B: Yes, because the amount in controversy does not exceed $75,000. C: No, because the illustrator's employment contract claim arises from the same transaction or occurrence as the federal employment law claim. D: No, because the federal court has personal jurisdiction over the corporation given that its headquarters are located in State B.

C is correct. Here, the court has subject-matter jurisdiction over the federal employment law claim and the contract claim arises from the same transaction or occurrence. Therefore, the court may hear the contract claim under supplemental jurisdiction. Under 28 U.S.C. § 1367, supplemental jurisdiction allows a federal court with original subject-matter jurisdiction to hear other claims arising out of the same transaction or occurrence as the original claim, even if they do not have independent grounds for federal jurisdiction. The underlying basis for subject-matter jurisdiction may be either federal question or diversity.

A senator from State A became upset when he learned that residents of State A are not permitted to bring state law products liability suits against State A companies in State A federal courts, but that residents of State B may do so. The senator plans to introduce legislation that states: "Federal courts shall have jurisdiction when the plaintiff's cause of action shows that it is based upon a state products liability law." Is the proposed legislation constitutional? A: Yes, because Congress has plenary power to extend federal jurisdiction to any questions arising under federal law or the U.S. Constitution. B: Yes, because Congress has plenary power to create federal courts and determine their jurisdiction. C: No, because it would extend federal jurisdiction to state law claims without requiring diversity of citizenship among the parties. D: No, because it would not extend federal jurisdiction to defendants who invoked state products liability as a defense.

C is correct. If the senator's proposed legislation were to pass, it would allow a plaintiff to bring suit and avoid the diversity requirement, which would be unconstitutional.

A husband from State A filed a divorce action against his wife, a citizen of State B, in State A federal court. The husband asked the court to divide the marital estate valued at $2,755,000. The couple lived in State B at all times during the marriage. The wife had never been to State A before the lawsuit was filed. The wife answered the husband's complaint without asserting any affirmative defenses or filing any additional motions. Two years into the lawsuit, after discovery had been completed and four days before trial, the wife filed a motion to dismiss, asserting that the court lacks subject-matter jurisdiction over the husband's claims. Should the court grant the wife's motion to dismiss? A: No, because her motion to dismiss was not timely. B: No, because she waived any affirmative defenses when she filed her answer. C: Yes, because the federal court in State A lacks subject-matter jurisdiction. D: Yes, because the district court in State A lacks personal jurisdiction.

C is correct. The U.S. Supreme Court has held that, even when the requirements for diversity of citizenship are satisfied, federal courts will not exercise jurisdiction over domestic relations or probate proceedings. Furthermore, lack of subject-matter jurisdiction may be asserted at any time and may not be waived.

A woman domiciled in a foreign country brought a wrongful discharge action in federal court in State A against her former employer. The employer is incorporated in State A and headquartered in State B. The woman had worked at the employer's office in the foreign country. Her on-site manager, who had sole discretion over personnel matters, had made the decision to discharge her. The manager has now retired and continues to live in the foreign country. Under the employment laws of the foreign country, if the woman succeeds in her action, her damages would be limited to two years' salary. If the woman succeeds in her action under State A employment law, her damages could be much greater and could include emotional distress and punitive damage. The employer filed a motion to dismiss the action for insufficient service of process, which the court denied. The employer then moved to dismiss the action for forum non conveniens. Is the court likely to dismiss the action for forum non conveniens? A: No, because the employer waived its right to challenge the forum by failing to include that challenge in its first motion to dismiss. B: No, because the remedies available to the woman under the foreign country's laws are less favorable than those available under State A law. C: Yes, because the foreign country is the more appropriate forum given that the discharge occurred there and the evidence is located there. D: Yes, because the woman is a foreign citizen and therefore her choice of forum is not entitled to deference.

C is correct. In deciding whether to dismiss an action for forum non conveniens, a court must weigh the private and public interests in keeping the action or dismissing it. Piper Aircraft Co. v. Reyno, 454 U.S. 235 (1981). Here, the foreign country has an interest in the resolution of the dispute because the plaintiff is one of its citizens and the conduct underlying the wrongful discharge claim took place there. Additionally, all the evidence and witnesses, including the plaintiff and the retired manager, are located in the foreign country, making it the most convenient place to try the action.

An investor from State A filed an action against his State B stockbroker in federal court in State A. The summons and complaint were served at the stockbroker's office in State B, where the process server handed the documents to the stockbroker's administrative assistant. The stockbroker has answered the complaint, asserting the defense of improper service of process. Assume that both states' requirements for service of process are identical to the requirements of the Federal Rules of Civil Procedure. Is the court likely to dismiss the action for improper service of process? A: No, because service was made on a person of suitable age found at the stockbroker's place of employment. B: No, because the stockbroker waived her claim for improper service of process by asserting it in her answer. C: Yes, because an individual defendant may not be served by delivering process to a third party found at the defendant's place of employment. D: Yes, because the process of State A courts is not effective in State B.

C is correct. No facts suggest that the administrative assistant was a designated agent of the stockbroker, and the Rules provide no general authority to serve process on third parties at a defendant's place of employment.

A teacher, a citizen of State A, sued a librarian, a citizen of State B, in a state court in State A. The teacher is seeking $100,000 as compensation for tortious injuries allegedly caused by the librarian's negligent acts in State A. The librarian filed a notice of removal in the federal district court of State B. The teacher filed a timely motion to remand to state court. Should the court grant the motion to remand? A: No, because the lawsuit was filed in State A, where the teacher is a citizen and the librarian is not. B: No, because the federal court has diversity jurisdiction over the parties and the original lawsuit was not filed in State B. C: Yes, because the original lawsuit was filed in State A. D: Yes, because the federal court does not have personal jurisdiction.

C is correct. Removal is only proper to the federal district where the original state court action is pending. Here, because the teacher filed the original lawsuit in State A, the only proper venue for removal would have been federal court in State A, not State B.

A state has a statute providing that an unsuccessful candidate in a primary election for a party's nomination for elected public office may not become a candidate for the same office at the following general election by nominating petition or by write-in votes. A woman sought her party's nomination for governor in the May primary election. After losing in the primary, the woman filed nominating petitions containing the requisite number of signatures to become a candidate for the office of governor in the following general election. The chief elections officer of the state refused to certify the woman's petitions solely because of the above statute. The woman then filed suit in federal district court challenging the constitutionality of this state statute. As a matter of constitutional law, which of the following is the proper burden of persuasion in this suit? A: The woman must demonstrate that the statute is not necessary to achieve a compelling state interest. B: The woman must demonstrate that the statute is not rationally related to a legitimate state interest. C: The state must demonstrate that the statute is the least restrictive means of achieving a compelling state interest. D: The state must demonstrate that the statute is rationally related to a legitimate state interest.

C is correct. Restrictions on the ability of individuals to be candidates must be examined to determine whether they violate the First Amendment freedoms regarding association and expression or the Fourteenth Amendment Equal Protection Clause. Here, First Amendment rights are involved in the action because the statute infringes on the right of political association. As a result, strict scrutiny is the appropriate level of review for the statute.

Insurance is provided in a particular state only by private companies. Although the state insurance commissioner inspects insurance companies for solvency, the state does not regulate their rates or policies. An insurance company charges higher rates for burglary insurance to residents of one part of a county in the state than to residents of another section of the same county because of the different crime rates in those areas. The plaintiff is a resident of the county who was charged the higher rate by the insurance company because of the location of her residence. The plaintiff sues the insurance company, alleging that the differential in insurance rates unconstitutionally denies her the equal protection of the law. Will the plaintiff's suit succeed? A: Yes, because the higher crime rate in the plaintiff's neighborhood demonstrates that the county police are not giving persons who reside there the equal protection of the laws. B: Yes, because the insurance rate differential is inherently discriminatory. C: No, because the constitutional guarantee of equal protection of the law is not applicable to the actions of these insurance companies. D: No, because there is a rational basis for the differential in insurance rates.

C is correct. The Constitution provides for equal protection of the law, which means that it protects individuals from actions by the state (the Fourteenth Amendment) or the federal government (the Fifth Amendment). Equal protection only restricts private action in extremely specific circumstances, none of which are present in this fact pattern.

A consumer brought a federal diversity action against a manufacturer, seeking damages for products liability claims. In its answer, the manufacturer included the affirmative defense of contributory negligence. Applicable state law had recently abolished contributory negligence as a defense in such actions. Before trial, the judge allowed the parties to submit proposed jury instructions. The manufacturer's attorney proposed an instruction that the jury should not return a verdict for the consumer if it found that the consumer had been contributorily negligent. After the close of the evidence, the judge told the parties that he would give the contributory negligence instruction. The consumer's attorney did not object. The judge instructed the jury. After the jury began their deliberations, the consumer's attorney objected to the contributory negligence instruction. Should the judge consider the objection? A: No, because the consumer's attorney did not object before the judge gave the instruction. B: No, because the consumer's attorney did not object when the judge gave the instruction. C: Yes, because the fact that the judge gave the inapplicable instruction constituted plain error that affected the consumer's substantial rights. D: Yes, because the consumer's attorney objected within 28 days after the judge gave the instruction.

C is correct. The applicable state law does not recognize contributory negligence as a defense to this kind of claim, so instructing the jury on contributory negligence and allowing that instruction to stand will affect the consumer's substantive rights by permitting the consumer to lose the case based on an erroneous statement of the law. The inclusion of the instruction thus constitutes plain error. Further, a plain error that affects substantive rights does not require the claim of error to be properly preserved, so the untimely objection is not outcome determinative. Fed. R. Evid. 103(e).

A plaintiff domiciled in State A brought a federal diversity negligence action in State A against a defendant domiciled in State B. The action was based on an accident that had occurred in State C. The defendant was personally served with process at her office in State B, which is located 50 miles from the State A federal courthouse. The defendant travels to State A once each year for a weeklong vacation but has no other State A contacts. The defendant answered, denying all allegations. One week later, the defendant filed an amended answer, denying all allegations and including the defense of lack of personal jurisdiction. State A has a long-arm statute that permits personal jurisdiction to the constitutional limit. The defendant has moved for an order dismissing the action based on the personal-jurisdiction challenge asserted in the amended answer. Should the court issue the order? A: No, because the defendant waived the challenge to personal jurisdiction by failing to include it in her original answer. B: No, because the defendant was personally served with process within 100 miles of the federal courthouse where the action is pending. C: Yes, because the defendant lacks minimum contacts with State A. D: Yes, because service was not delivered to the defendant at her home.

C is correct. The defendant lacks sufficient minimum contacts with State A that are related to this lawsuit. The defendant is not domiciled in State A and only visits periodically for reasons unrelated to the plaintiff's claim.

A man who owned riverfront property sued an upstream factory in federal court for polluting the river, seeking injunctive relief and $250,000 in damages. The factory moved for summary judgment on the ground of res judicata (claim preclusion), arguing that the man had sued on and lost an identical claim one year before. The court denied the motion. The factory has asked its attorney's advice as to whether it may appeal the court's denial of summary judgment in order to avoid an expensive trial. What advice should the attorney give? A: The factory may appeal if the appellate court finds that the case involves a controlling question of law upon which the courts are divided. B: The factory may appeal if the trial court certifies that there is no just reason for delay. C: The factory may not appeal until after a trial on the merits or other disposition resulting in a final judgment. D: The factory may not appeal, because the denial of summary judgment is a collateral order.

C is correct. The denial of a motion for summary judgment is generally not appealable until after a trial on the merits, or a final judgment.

An accountant from State A sued a biologist from State A in a federal court in State A. The accountant did not state a claim based on federal law in her complaint. However, the accountant believes the biologist will use a federal law in his defense. Should the court dismiss the accountant's complaint? A: No, because the judge should wait and determine if any pleadings, filed by either side, contain a federal question. B: No, because the judge should wait to see whether the biologist's answer contains a defense based on a federal question. C: Yes, because the complaint does not contain a federal question. D: Yes, because the complaint did not state the anticipated defense based in federal law.

C is correct. The federal question establishing subject-matter jurisdiction must appear as part of the accountant's cause of action as set out in a well-pleaded complaint. The fact that the biologist's defense might be based in federal law is irrelevant and does not establish federal question jurisdiction over the accountant's claim. The court may not look to a defense asserted by the biologist to determine whether the accountant's complaint raises a federal question.

A neighbor sued a man in state court, challenging the man's use of an implied easement on the neighbor's property. The neighbor claimed the implied easement was invalid. The state court found for the man, holding that the easement was valid on the grounds of longstanding use. Thereafter, the neighbor sold the property to a buyer. The buyer refused to recognize the easement. The man then sued the buyer in state court, seeking a declaration that the implied easement on the buyer's property was valid. The man's attorney filed a motion for summary judgment. Is the court likely to grant the man's motion for summary judgment? A: No, because the buyer waived the defense of claim preclusion by failing to file a motion to dismiss. B: No, because the buyer was not a party in the prior lawsuit. C: Yes, because the man should prevail on the ground of issue preclusion. D: Yes, because the man should prevail on the ground of claim preclusion.

C is correct. The neighbor and buyer are in privity because they were successive owners of the same property. Thus, the buyer is bound by the final judgment on the issue of the validity of the easement in the prior case and the court should find in favor of the man by granting his motion for summary judgment.

The number of large billboards along a city's major roadways has increased considerably in recent years. Many political groups have contributed to that increase with their growing use of billboards to display messages that further a variety of political candidates and public causes. The city council has received many complaints from residents claiming that the billboards are ugly and detract from picturesque sights along the roadways. In response to these complaints, the city council has enacted an ordinance that substantially limits the number and the size of billboards that may be erected along designated scenic roadways. A company that owns many of the billboards affected by this ordinance has filed suit, claiming that the ordinance violates the company's freedom of speech. Should the court uphold the ordinance? A: No, because the ordinance burdens political speech and is not necessary to further a compelling government interest. B: No, because the ordinance restricts speech in a traditional public forum. C: Yes, because the ordinance is narrowly tailored to further the substantial government interest in the city's appearance. D: Yes, because the ordinance reasonably regulates the use of public roadways.

C is correct. The ordinance is a content-neutral restriction on speech because it regulates without regard to the content of the message expressed on the billboards. Content-neutral speech restrictions receive intermediate judicial scrutiny, which requires the restriction to be narrowly tailored to further a substantial government interest. The Supreme Court has held that the aesthetic interests of a community are sufficiently substantial to justify content- neutral restrictions on public displays of signs. See Metromedia, Inc. v. City of San Diego, 453 U.S. 490 (1981). The ordinance here is narrowly tailored because it applies only to scenic roadways, and it does not completely ban signs.

A plaintiff filed a tort action in state court but then failed to prosecute the action. The defendant moved to dismiss the action, and the court granted the motion in an order that stated: "The defendant's motion to dismiss is granted, and this action is dismissed with prejudice." The court accordingly entered judgment for the defendant. The plaintiff then filed the same claim against the defendant in federal court, invoking diversity jurisdiction. The defendant has asserted the defense of res judicata (claim preclusion) in its answer. Should the federal court give preclusive effect to the state court judgment? A: No, because the judgment was entered by a state court, not a federal court. B: No, because the state court did not rule on the merits in its dismissal. C: Yes, because a dismissal with prejudice operates as a judgment on the merits. D: Yes, because a judgment for failure to prosecute operates as a judgment on the merits under the Federal Rules of Civil Procedure.

C is correct. The state court decision operates as a final judgment on the merits under Federal Rule of Civil Procedure (FRCP) 41(b) and claim preclusion consequently bars the claim from being brought again.

A state statute declares that after five years of continuous service in their positions, all state employees, including faculty members at the state university, are entitled to retain their positions during "good behavior." The statute also contains a number of procedural provisions. Any state employee who is dismissed after that five-year period must be given reasons for the dismissal before it takes effect. In addition, such an employee must, upon request, be granted a post-dismissal hearing before an administrative board to seek reinstatement and back pay. The statute precludes any other hearing or opportunity to respond to the charges. That post-dismissal hearing must occur within six months after the dismissal takes effect. The burden of proof at such a hearing is on the state, and the board may uphold the dismissal only if it is supported by a preponderance of the evidence. An employee who is dissatisfied with a decision of the board after a hearing may appeal its decision to the state courts. The provisions of this statute are inseverable. A teacher who had been employed continuously for seven years as a faculty member at the state university was dismissed. A week before the dismissal took effect, she was informed that she was being dismissed because of a charge that she accepted a bribe from a student in return for raising the student's final grade in her course. At that time she requested an immediate hearing to contest the propriety of her dismissal. Three months after her dismissal, she was granted a hearing before the state administrative board. The board upheld her dismissal, finding that the charge against her was supported by a preponderance of the evidence presented at the hearing. The faculty member did not appeal the decision of the state administrative board to the state courts. Instead, she sought a declaratory judgment in federal district court to the effect that the state statute prescribing the procedures for her dismissal is unconstitutional. In this case, the federal district court should A: dismiss the suit, because a claim that a state statute is unconstitutional is not ripe for adjudication by a federal court until all judicial remedies in state courts provided for by state law have been exhausted. B: hold the statute unconstitutional because the Due Process Clause of the Fourteenth Amendment requires a state to demonstrate beyond a reasonable doubt the facts constituting good cause for termination of a state employee. C: hold the statute unconstitutional, because a state may not ordinarily deprive an employee of a property interest in a job without giving the employee a pre-termination opportunity to respond to the charges against that employee. D: hold the statute constitutional, because the Due Process Clause of the Fourteenth Amendment entitles state employees who have a right to their jobs during good behavior only to a statement of reasons for their dismissal and an opportunity for a post-dismissal hearing.

C is correct. The teacher's procedural due process rights have been violated because of the deprivation of her property interest in continued public employment. She had the right to receive notice of her termination and a pre- termination hearing with an opportunity to respond. However, she was not given a hearing until after her termination was effective, and therefore, the statute is unconstitutional.

Public schools in a state are financed, in large part, by revenue derived from real estate taxes imposed by each school district on the taxable real property located in that district. Public schools also receive other revenue from private gifts, federal grants, student fees, and local sales taxes. For many years, the state has distributed additional funds, which come from the state treasury, to local school districts in order to equalize the funds available on a per-student basis for each public school district. These additional funds are distributed on the basis of a state statutory formula that considers only the number of students in each public school district and the real estate tax revenue raised by that district. The formula does not consider other revenue received by a school district from different sources. The school boards of two school districts, together with parents and schoolchildren in those districts, bring suit in federal court to enjoin the state from allocating the additional funds from the state treasury to this formula. They allege that the failure of the state, in allocating this additional money, to take into account a school district's sources of revenue other than revenue derived from taxes levied on real estate located there violates the Fourteenth Amendment. The complaint does not allege that the allocation of the additional state funds based on the current statutory formula has resulted in a failure to provide minimally adequate education to any child. Which of the following best describes the appropriate standard by which the court should review the constitutionality of the state statutory funding formula? A: Because classifications based on wealth are inherently suspect, the state must demonstrate that the statutory formula is necessary to vindicate a compelling state interest. B: Because the statutory funding formula burdens the fundamental right to education, the state must demonstrate that the formula is necessary to vindicate a compelling state interest. C: Because no fundamental right or suspect classification is implicated in this case, the plaintiffs must demonstrate that the funding allocation formula bears no rational relationship to any legitimate state interest. D: Because the funding formula inevitably leads to disparities among the school districts in their levels of total funding, the plaintiffs must only demonstrate that the funding formula is not substantially related to the furtherance of an important state interest.

C is correct. There is no fundamental right to education, and wealth is not a suspect class. Because there is no fundamental right or suspect class at issue, the court will apply rational basis review, which would require the plaintiffs to demonstrate that the funding allocation formula is not rationally related to any conceivable legitimate state interest.

On July 29, 2012, a car manufacturer filed a lawsuit against a corporation in federal court. The manufacturer mistakenly believed that the corporation was the entity responsible for producing the bolts used to secure the car tires onto the car manufacturer's cars. The manufacturer was sued after some of the cars' tires became detached during transit due to faulty bolts. The manufacturer is seeking contractual indemnification for losses stemming from these lawsuits. State law provides a five-year statute of limitations for contract claims, which starts running when a plaintiff first learns of its contract claim. The manufacturer learned of the faulty bolts on August 1, 2007, and realized it might have a contract claim. Two weeks after filing its lawsuit, the manufacturer realized it had mistaken the corporation for a different entity that was, in fact, responsible for producing the faulty bolts. The other entity not only knew the bolts were faulty, but was also aware of the original lawsuit against the corporation. The other entity was hoping that the manufacturer would not realize its mistake. Is the court likely to allow the manufacturer to substitute the other entity as a defendant? A: No, because it would be unconstitutional to allow the other entity to be made a defendant after the statute of limitations has expired. B: No, because the Federal Rules of Civil Procedure do not permit a party to be added to a lawsuit after the statute of limitations has expired. C: Yes, because the other entity knew about the manufacturer's original lawsuit and that it would have been sued but for the mistake concerning the party's identity. D: Yes, because the claim against the other entity arises out of the conduct, transaction, or occurrence set out in the manufacturer's original complaint.

C is correct. Under Federal Rule of Civil Procedure (FRCP) 15(c)(1)(C), an amendment to a pleading to change a party relates back to the date the original pleading was filed for purposes of the statute of limitations if: (i) the claim arises out of the conduct, transaction, or occurrence set out - or attempted to be set out - in the original pleading; AND (ii) if the new party received notice of the plaintiff's original lawsuit and knew, or should have known about the action but-for a mistake concerning the proper party's identity.

A law of a state imposed a generally applicable sales tax payable by the vendor. That law exempted from its provisions the sale of "all magazines, periodicals, newspapers, and books." In order to raise additional revenue, the state legislature eliminated that broad exemption and substituted a narrower exemption. The new, narrower exemption excluded from the state sales tax only the sale of those "magazines, periodicals, newspapers, and books that are published or distributed by a recognized religious faith and that consist wholly of writings sacred to such a religious faith." One affected magazine is a monthly publication devoted to history and politics. The magazine paid under protest the sales tax due on its sales according to the amended sales tax law. The magazine then, in federal district court, brought a challenge to the constitutionality of the statute and requested damages for the sales taxes paid. It contended that the new, narrower exemption restricted to sacred writings of recognized religious faiths violates the First and Fourteenth Amendments to the Constitution. Assume that the magazine has proper standing. In this case, the court will probably rule that A: the new, narrower exemption from the state sales tax law violates substantive due process because it is a deprivation of property and thus amounts to a taking from the magazine. B: the new, narrower exemption from the state sales tax law violates the Privileges or Immunities Clause of the Fourteenth Amendment because it denies the magazine the same rights granted to publications of recognized religious faiths. C: the new, narrower exemption from the state sales tax law violates the Establishment Clause of the First and Fourteenth Amendments by granting preferential state support to recognized religious faiths for the communication of their religious beliefs. D: the new, narrower exemption from the state sales tax law violates the freedom of the press guaranteed by the First and Fourteenth Amendments because it imposes a prior restraint on nonreligious publications that are required to pay the tax.

C is correct. Under the Establishment Clause, the government cannot pass a law that formally sponsors or establishes a religion. The new sales tax exemption gives a benefit only to "recognized religious faiths" and only for the publication of their sacred writings. That exemption is effectively a subsidy of recognized religious faiths. Therefore, it violates the Establishment Clause.

A traveler filed a negligence suit in federal court against a bus driver for injuries he sustained from an accident with the driver. In actuality, the person driving the bus when the accident occurred was the driver's twin sister. Service of the complaint naming the driver as the defendant was timely made on the twin sister, who forwarded the complaint to the driver's liability insurer. The insurer provided verification to the traveler's attorney that the named defendant, the driver, was in another state at the time of the accident, and the bus was being driven by the twin sister. Thereafter, the statute of limitations on the traveler's claim expired. The court granted the traveler leave to amend his complaint to change the named defendant to the twin sister, who moved to dismiss based on the statute of limitations having expired. Should the court grant the twin sister's motion to dismiss? A: Yes, because the court improperly granted leave to amend the original complaint after the statute of limitations had run. B: Yes, because there the statute of limitations is a defense that is apparent based on the face of the complaint. C: No, because the amended complaint relates back to the date the original complaint was filed. D: No, because the original defendant's name was a misnomer, and changing it was permitted as a matter of course.

C is correct. Under the Federal Rules of Civil Procedure (FRCP), an amendment that changes the party against whom a claim is asserted relates back to the date of the original complaint. It does so when the amendment asserts a claim that arose out of the conduct set out in the original complaint and, within the period provided for serving the summons and complaint, the party to be brought in received such notice of the action that it will not be prejudiced in defending on the merits and knew or should have known that the action would have been brought against it, but for a mistake concerning the proper party's identity.

A former employee sued a corporation, her former employer, in State A state court, alleging that she was illegally fired after bringing a worker's compensation claim. The employee is a citizen of State B and the corporation is incorporated in State C, with its principal manufacturing plant in State B, its second-largest plant in State A, and its board of directors located in State D. The employee is seeking over $75,000 in damages. Under State A's worker's compensation law, as interpreted by State A courts, this claim "arises under" State A worker's compensation law. The corporation removed the case to the federal district court in State A, invoking diversity jurisdiction. Within 30 days after the filing of the notice of removal, the employee moved to remand back to state court. Should the court grant the employee's motion to remand? A: No, because the court has subject-matter jurisdiction based on diversity. B: No, because the employee's motion to remand was untimely. C: Yes, because a worker's compensation action in state court, arising under state law, may not be removed. D: Yes, because the court lacks subject-matter jurisdiction over the case.

C is correct. Under the federal removal statute, certain actions are nonremovable. A civil action in any state court arising under the worker's compensation law of that state may not be removed to any federal district court, regardless of whether diversity jurisdiction exists.

A bakery incorporated and headquartered in State A had a dispute with a mill incorporated and headquartered in State B over the quality of the flour the mill had delivered to the bakery. The bakery sued the mill in a federal court in State A for breach of contract, seeking $100,000 in damages. The contract between the bakery and the mill contained a clause designating State B courts as the sole venue for litigating disputes arising under the contract. Under precedent of the highest court in State A, forum-selection clauses are unenforceable as against public policy; under U.S. Supreme Court precedent, such clauses are enforceable. The mill has moved to transfer the case to a federal court in State B, citing the forum-selection clause in the parties' contract and asserting the facts that the flour was produced in State B and that the majority of likely witnesses are in State B. Is the court likely to grant the mill's motion? A: No, because State A law treats forum- selection clauses as unenforceable. B: No, because the mill should have instead filed a motion to dismiss for improper venue. C: Yes, because federal common law makes the forum-selection clause controlling. D: Yes, because federal law governs transfers of venue, and it would be more convenient for the witnesses and parties to litigate the claim in State B.

D is correct. A forum-selection clause represents the parties' agreement as to the most proper forum, and it will be enforced by means of a motion to transfer "in the interest of justice" under 28 U.S.C. § 1404. Here, likely most of the witnesses and evidence are in State B, so the court should uphold the clause and transfer for convenience.

A tenant living in State A sued a landlord from State B in a single count complaint filed in the federal district court of State A. The complaint alleged that while the tenant was living in an apartment building owned by the landlord in State B, the landlord breached his duty to keep the premises secure and a break-in occurred, causing the tenant $90,000 in property damages. Ten years before the tenant filed the lawsuit, the landlord inherited a small farm in State A from his father. The landlord has never visited the farm or been physically present in State A. The landlord's farm is valued at $50,000. May the State A federal court hear the action? A: Yes, because the parties are diverse and the landlord owns property in State A. B: Yes, because the parties are diverse and the value of the landlord's property exceeds $75,000. C: No, because the value of the farm is less than the tenant's alleged damages. D: No, because the property has no relationship to the lawsuit and the landlord has no other contacts with State A.

D is correct. Although the court has subject-matter jurisdiction based on diversity, it does not have personal jurisdiction over the landlord. Owning land that is entirely unrelated to the subject of the underlying suit, with no other contacts in State A, prevents the court from constitutionally exercising personal jurisdiction over the landlord.

Two ranchers, both citizens of State A, brought an action in a state court in State A against a developer, a citizen of State B. The ranchers alleged a state-law tort claim for water runoff damage to their properties caused by construction on the developer's neighboring property. The first rancher claimed $250,000 in damages and the second rancher claimed $50,000. In their complaint, the ranchers cited federal law regarding the calculation of damages due to water runoff. The developer timely removed the action to federal court. Is removal proper? A: No, because the ranchers are not diverse from each other. B: No, because the second rancher's claim does not meet the amount-in-controversy requirement. C: Yes, because the complaint includes a federal question. D: Yes, because the ranchers are diverse from the developer and both ranchers' claims arise from the same facts.

D is correct. Although the second rancher's claim does not meet the amount-in-controversy minimum for federal diversity jurisdiction, supplemental jurisdiction authorizes jurisdiction over claims that otherwise would not meet the amount-in-controversy requirement. See 28 U.S.C. § 1367; Exxon Mobil Corp. v. Allapattah Services, 545 U.S. 546 (2005). To establish supplemental jurisdiction, the insufficient claims must be so related to claims in the action that are within the court's original jurisdiction that they form part of the "same case or controversy." Here, both ranchers' claims arise from water runoff caused by the same construction on the neighboring property and thus meet that standard.

A patient, a citizen of State A, sued a physician, a citizen of State B, in a federal court in State B. The patient is seeking $125,000 as compensation for medical malpractice. The only allegation concerning the physician's misconduct in the patient's complaint is that the physician "practiced medicine negligently." The patient's complaint contains no further explanation of the physician's alleged negligence. Is the court likely to find the patient's complaint sufficient? A: Yes, because it satisfies federal pleading requirements, though the allegations alone would not survive a motion for summary judgment. B: Yes, because it satisfies State A's pleading requirements, which are applicable in this diversity action. C: No, because it does not satisfy federal pleading requirements and may be subject to a motion for summary judgment. D: No, because it does not satisfy federal pleading requirements and may be subject to a motion to dismiss.

D is correct. As a federal procedural issue, this complaint is governed by FRCP notice pleading. By merely stating that the physician "practiced medicine negligently," the patient's complaint does not satisfy the requirement that pleadings must contain enough facts to show that the claim is plausible, not merely possible. An insufficient complaint is vulnerable to a motion to dismiss.

An engineer brought a patent infringement action against a company in federal court in State A. The case was tried by a jury. During examination, the engineer's attorney asked one of the company's representatives whether the representative's cousin was a "bottom-feeder who swims around buying, for next to nothing, the houses of people who got kicked out." The court warned the engineer's counsel against making such inflammatory remarks and instructed the jury to ignore such statements. In his closing argument, the engineer's attorney again made several irrelevant and prejudicial remarks. The company's attorney objected to the remarks and moved for a new trial. This was the first time the attorney objected to irrelevant and prejudicial remarks. How is the court likely to rule on the motion for a new trial? A: Deny the motion, because juries are expected to identify patently biased statements and disregard them. B: Deny the motion, because of the company's counsel's failure to object, prior to closing arguments, to the opposing counsel's irrelevant and prejudicial remarks. C: Deny the motion, because instructions for the jury to disregard the inflammatory statements as non- evidence cures any harm otherwise done. D: Grant the motion, because the jury is likely to be biased after hearing these inflammatory remarks, and manifest injustice would otherwise result.

D is correct. It is likely that the jury would be biased by the repetitive derogatory comments from the engineer's attorney, and manifest injustice would result from letting its future verdict stand.

A plaintiff domiciled in State A has brought a federal diversity action in State A against a defendant domiciled in State B, seeking damages for injuries the defendant allegedly caused the plaintiff in State B. The defendant has never been to State A and has no connections there. The defendant's attorney knows that the defendant was properly served but does not believe that the State A court has personal jurisdiction over her. What is the best way for the attorney to raise the argument that the court lacks personal jurisdiction over the defendant? A: Move for discovery on the issue of personal jurisdiction. B: Move for judgment on the pleadings, seeking dismissal of the action for lack of personal jurisdiction. C: Move for sanctions against the plaintiff and his attorney for filing the action in a court that has no personal jurisdiction over the defendant. D: Move to dismiss the action for lack of personal jurisdiction based on the complaint allegations and an affidavit from the defendant about her lack of connection to State A.

D is correct. Some defenses must be raised by the defendant the first time he files a motion or in his answer, whichever is first, or else the defenses are waived. Lack of personal jurisdiction is one of these defenses that must be filed at the defendant's first motion or answer or else it will be waived. In this question, the attorney should raise a motion to dismiss for lack of personal jurisdiction first before any other filings with the court.

In order to foster an environment conducive to learning, a school board enacted a dress code that prohibited all public high school students from wearing in school shorts cut above the knee. Because female students at the school considered it unfashionable to wear shorts cut at or below the knee, they no longer wore shorts to school. On the other hand, male students at the school regularly wore shorts cut at or below the knee because they considered such shorts to be fashionable. Female students sued to challenge the constitutionality of the dress code on the ground that it denied them the equal protection of the laws. Should the court uphold the dress code? A: No, because the dress code is not necessary to further a compelling state interest. B: No, because the dress code is not substantially related to an important state interest. C: Yes, because the dress code is narrowly tailored to further an important state interest. D: Yes, because the dress code is rationally related to a legitimate state interest.

D is correct. The court should uphold the dress code because it is rationally related to the state's legitimate interest in fostering a proper educational environment. The dress code should not trigger heightened judicial scrutiny because there are no facts to suggest that the purpose of the code is to discriminate against female students.

The legislature of a particular state enacted a statute requiring that all law enforcement officers in that state be citizens of the United States. An alien, lawfully admitted to permanent residency five years before the enactment of this statute, sought employment as a forensic pathologist in the state coroner's office. He was denied such a job solely because he was not a citizen. The alien thereupon brought suit in federal district court against appropriate state officials seeking to invalidate this citizenship requirement on federal constitutional grounds. The strongest ground upon which to attack this citizenship requirement is that it A: constitutes an ex post facto law as to previously admitted aliens. B: deprives an alien of a fundamental right to employment without the due process of law guaranteed by the Fourteenth Amendment. C: denies an alien a right to employment in violation of the Privileges or Immunities Clause of the Fourteenth Amendment. D: denies an alien the equal protection of the laws guaranteed by the Fourteenth Amendment.

D is correct. The Equal Protection Clause requires that classifications based on alienage be narrowly tailored to promote a compelling state interest. However, strict scrutiny for alienage classifications does not apply where the discrimination against aliens relates to functions that go to the heart of the representative government. This means that individuals who hold state-elected or important non-elected executive positions, and those who are government officers that participate directly in the formulation, execution, or review of broad public policy may be required to be citizens. If the law discriminates against an alien participating in state government, the standard is rational basis review. It has been held that the government may discriminate against aliens with respect to the following positions: state troopers; public school teachers; jurors; and deputy probation officers. In this case, it is unclear which standard the position of forensic pathologist would be evaluated against. However, because this law is treating a group of people differently, the best challenge would be under equal protection.

A city owns and operates a large public auditorium. It leases the auditorium to any group that wishes to use it for a meeting, lecture, concert, or contest. Each user must post a damage deposit and pay rent, which is calculated only for the actual time the building is used by the lessee. Reservations are made on a first-come, first-served basis. A private organization that permits only males to serve in its highest offices rented the auditorium for its national convention. The organization planned to install its new officers at that convention. It broadly publicized the event, inviting members of the general public to attend the installation ceremony at the city auditorium. No statute or administrative rule prohibits the organization from restricting its highest offices to men. An appropriate plaintiff sues the private organization seeking to enjoin it from using the city auditorium for the installation of its new officers. The sole claim of the plaintiff is that the use of this auditorium by the organization for the installation ceremony is unconstitutional because the organization disqualifies women from serving in its highest offices. Will the plaintiff prevail? A: Yes, because the Fourteenth Amendment prohibits such an organization from discriminating against women in any of its activities to which it has invited members of the general public. B: Yes, because the organization's use of the city auditorium for this purpose subjects its conduct to the provisions of the Fourteenth Amendment. C: No, because the freedom of association protected by the Fourteenth Amendment prohibits the city from interfering in any way with the organization's use of city facilities. D: No, because the organization is not a state actor and, therefore, its activities are not subject to the provisions of the Fourteenth Amendment.

D is correct. The Fourteenth Amendment prohibits state action but not private action. Therefore, the prohibition of women from serving as officers in this organization is not unconstitutional.

A beneficiary of a trust, who is a citizen of State A, has sued the trustee in federal court in State A for failing to correctly distribute the income from the trust, seeking an accounting. The trustee was personally served with process and the complaint by the beneficiary's attorney while the trustee was vacationing in State A. The trustee is a citizen of State B, and the accounts that are the subject of the trust are located in State B. What is the trustee's best response to the complaint? A: Answer the complaint and counterclaim for abuse of process. B: File an action in a State B court and move to enjoin the State A action. C: Move to dismiss for improper service by the attorney. D: Move to dismiss for lack of personal jurisdiction.

D is correct. The court in State A lacks personal jurisdiction over the trustee. Although the trustee was served in State A, the facts fail to indicate any other contacts between the trustee and State A, and the trust assets are located in State B. In addition, the trustee was sued in a representative capacity but was served while in State A on a personal vacation. Thus, the trustee can argue that the service did not establish personal jurisdiction over him concerning his role as trustee. Further, the trustee has no minimum contacts as a trustee with State A to satisfy the requirements of fair play and substantial justice. See Hanson v. Denkla, 357 U.S. 235 (1958).

A pilot from State A sued a librarian from State B in a federal district court in State A. The librarian sold the pilot an expensive collectible watch through a website that generated a valid sales contract. The pilot's complaint alleged that the librarian breached the contract's terms because the librarian sold the pilot a watch that materially varied from the description the librarian had provided. The pilot is seeking $175,000 in damages. Shortly after selling the watch, the librarian went to State A on vacation with her family. This was the librarian's first time ever in State A. While visiting State A, the librarian was served with the pilot's complaint and a summons in accordance with State A law. The librarian filed a motion to dismiss for lack of jurisdiction. Should the court grant the librarian's motion to dismiss? A: Yes, because only federal courts have jurisdiction to hear disputes between citizens of different states with an amount in controversy over $75,000. B: Yes, because the librarian does not have sufficient minimum contacts with State A to establish personal jurisdiction. C: No, because personal jurisdiction requirements only apply to federal courts, not state courts. D: No, because the librarian's physical presence in State A when she was served is sufficient to establish personal jurisdiction.

D is correct. The court should not grant the motion because both subject-matter and personal jurisdiction are satisfied. The facts support diversity jurisdiction based on the citizenship of the parties and amount in controversy. Furthermore, under Pennoyer v. Neff, 95 U.S. 714 (1878), a court may exercise personal jurisdiction over a non-resident defendant if she is present in the forum state when personally served with process. This occurred here, when the librarian was in State A on vacation and personally served with the complaint and summons in accordance with State A law.

A biker filed a lawsuit against a driver in state court in State A, seeking compensation for damages incurred in a collision. The driver defended on the ground of contributory negligence, a full defense under State A law. The jury rendered a general verdict for the driver, thereby not identifying its specific findings. A doctor, who was also injured in the same collision, subsequently filed a jurisdictionally valid diversity lawsuit against the driver in State A federal court. Will preclusion prevent litigation of the negligence claim in the doctor's lawsuit? A: Yes, the driver will be precluded from relitigating the issue of his negligence because of the first lawsuit. B: Yes, the doctor will be precluded from relitigating the issue of the driver's negligence because of the first lawsuit. C: No, issue preclusion will not apply because of the first lawsuit, but claim preclusion may apply. D: No, neither issue preclusion nor claim preclusion will apply because of the first lawsuit.

D is correct. The doctor's lawsuit against the driver will not be subject to issue or claim preclusion. It is unclear in the first lawsuit what the jury decided, and the doctor was not a party in, nor was he in privity with any party from, the first lawsuit.

Members of a religious group calling itself the Friends of Lucifer believe in Lucifer as their Supreme Being. The members of this group meet once a year on top of Mt. Snow, located in a U.S. National Park, to hold an overnight encampment and a midnight dance around a large campfire. They believe this overnight encampment and all of its rituals are required by Lucifer to be held on top of Mt. Snow. U.S. National Park Service rules that have been consistently enforced prohibit all overnight camping and all campfires on Mt. Snow because of the very great dangers overnight camping and campfires would pose in that particular location. As a result, the park Superintendent denied a request by the Friends of Lucifer for a permit to conduct these activities on top of Mt. Snow. The park Superintendent, who was known to be violently opposed to cults and other unconventional groups had, in the past, issued permits to conventional religious groups to conduct sunrise services in other areas of that U.S. National Park. The Friends of Lucifer brought suit in Federal Court against the U.S. National Park Service and the Superintendent of the park to compel issuance of the requested permit. As a matter of constitutional law, the most appropriate result in this suit would be a decision that denial of the permit was A: invalid, because the Free Exercise Clause of the First Amendment prohibits the Park Service from knowingly interfering with religious conduct. B: invalid, because these facts demonstrate that the action of the Park Service purposefully and invidiously discriminated against the Friends of Lucifer. C: valid, because the Establishment Clause of the First Amendment prohibits the holding of religious ceremonies on federal land. D: valid, because religiously motivated conduct may be subjected to nondiscriminatory time, place, and manner restrictions that advance important public interests.

D is correct. The government may regulate the time, place, and manner of religiously-motivated conduct as long as the regulation is neutral and serves an important public interest. Here, the regulation is neutral on its face and in application, the Superintendent's views notwithstanding. The interest in public safety is important. Therefore, the denial of the permit would be valid.

An interstate bus company operates in a five-state area. A federal statute authorizes the Interstate Commerce Commission (ICC) to permit interstate carriers to discontinue entirely any unprofitable route. The interstate bus company applied to the ICC for permission to drop a very unprofitable route through the sparsely populated Shaley Mountains. The ICC granted that permission even though the interstate bus company provided the only public transportation into the region. A man is the owner of a mountain resort in the Shaley Mountains, whose customers usually arrived on vehicles operated by the interstate bus company. After exhausting all available federal administrative remedies, the man filed suit against the interstate bus company in the trial court of the state in which the Shaley Mountains are located to enjoin the discontinuance by the interstate bus company of its service to that area. The man alleged that the discontinuance of service by the interstate bus company would violate a statute of that state prohibiting common carriers of persons from abandoning service to communities having no alternate form of public transportation. The state court should A: dismiss the action, because the man lacks standing to sue. B: direct the removal of the case to federal court, because this suit involves a substantial federal question. C: hear the case on its merits and decide for the man because, on these facts, a federal agency is interfering with essential state functions. D: hear the case on its merits and decide for the interstate bus company, because a valid federal law preempts the state statute on which the man relies.

D is correct. The man is suing under a state statute that forbids carriers from discontinuing routes to areas with no alternative means of public transit. This directly conflicts with a federal statute that authorizes transit companies to discontinue routes that are not profitable. Thus, the federal statute will preempt the state statute under the Supremacy Clause.

An accounting firm brought a federal diversity action against a former client for failing to pay for the firm's audit of the client's financial statements. After the client answered, the parties settled, and the court dismissed the action with prejudice. The client subsequently sued the firm for negligently performing the audit. The firm moved to dismiss the negligence action on the basis of res judicata (claim preclusion). Is the court likely to grant the motion? A: No, because the firm's negligence was never raised or decided in the first action. B: No, because the first action was resolved by settlement. C: Yes, because the court dismissed the first action with prejudice. D: Yes, because the negligence claim was transactionally related to the claim in the first action and should have been asserted as a counterclaim.

D is correct. The resolution of the first action precludes the client from asserting any claims arising out of the transaction that was the subject of that action. The negligence claim arose out of the same audit that was the subject of the dispute in the first action. The parties in the second action were adversaries in the first action, and that action was resolved through a settlement and dismissal with prejudice. Additionally, the client should have asserted the negligence claim in the first action as a compulsory counterclaim, and its failure to do so means that the court is likely to grant the motion to dismiss.

A retailer brought a federal diversity action against an architect, alleging fraudulent misrepresentations in the architect's design of the retailer's store. The complaint did not include a jury demand. The architect timely moved to dismiss the action for failure to state a claim; he did not file an answer. Twenty days after being served with the motion, the retailer amended the complaint to add a defamation claim based on the architect's recent statements about the retailer in a local newspaper. In the amended complaint, the retailer demanded a jury trial on both claims. Has the retailer properly demanded a jury trial? A: No, because the retailer filed the demand more than 14 days after service of the motion to dismiss. B: No, because the retailer filed the demand more than 14 days after service of the original complaint. C: Yes, but on the defamation claim only, because the original complaint did not contain a jury demand. D: Yes, on both claims, because the architect had not answered the original complaint when the retailer filed the amended complaint with the jury demand.

D is correct. The retailer did not miss the 14-day deadline and may still demand a jury because the architect did not file an answer to the first or second complaint.

Congress enacted a statute authorizing the denial of all federal funding to public school districts in which a specified percentage of the students enrolled in the public schools fail to pass a national achievement test. According to the terms of the federal statute, the first national achievement test was scheduled for administration five years from the effective date of the statute. After reviewing then-current levels of public school student performance, the officials of a state became concerned that several of its public school districts would lose their federal funding after the administration of the first national achievement test. Then-current levels of private school student performance were substantially higher. In order to improve the chances of those school districts retaining their federal funding, the state recently enacted a law that requires all children of elementary and secondary school age to attend the schools operated by their respective local public school districts. The law is to take effect at the beginning of the next school year. Parents of children enrolled in private schools within the state have filed suit to challenge the constitutionality of this state law. Should the court uphold the law? A: Yes, because it is rationally related to a legitimate state interest. B: Yes, because it is necessary to further a compelling state interest. C: No, because it is not rationally related to a legitimate state interest. D: No, because it is not necessary to further a compelling state interest.

D is correct. The state's law requiring kids to attend public schools defies the Court's holding in Pierce that such laws are invalid. Moreover, the law infringes upon parents' fundamental right to raise their children, which triggers strict scrutiny, a level of review that cannot be satisfied here.

A city ordinance requires every operator of a taxicab in the city to have a license and permits revocation of that license only for "good cause." The city taxicab operator's licensing ordinance conditions the issuance of such a license on an agreement by the licensee that the licensee "not display in or on his or her vehicle any bumper sticker or other placard or sign favoring a particular candidate for any elected municipal office." The ordinance also states that it imposes this condition in order to prevent the possible imputation to the city council of the views of its taxicab licensees and that any licensee who violates this condition shall have his or her license revoked. One holder of a city taxicab operator's license decorates his cab with bumper stickers and other signs favoring specified candidates in a forthcoming election for municipal offices. A proceeding is initiated against him to revoke his taxicab operator's license on the sole basis of that admitted conduct. In this proceeding, does this license holder have a meritorious defense based on the United States Constitution? A: No, because he accepted the license with knowledge of the condition and, therefore, has no standing to contest it. B: No, because a taxicab operator's license is a privilege and not a right and, therefore, is not protected by the Due Process Clause of the Fourteenth Amendment. C: Yes, because such a proceeding threatens the license holder with a taking of property, his license, without just compensation. D: Yes, because the condition imposed on taxicab operators' licenses restricts political speech based wholly on its content, without any adequate governmental justification.

D is correct. There is a meritorious defense based on the freedom of expression because the licensing restriction is content-based (prohibiting political candidate bumper stickers on private property) and as such, it is subject to strict scrutiny. The city's interest in preventing the possibility of people thinking taxi drivers and councilmembers have the same political views has not been shown to be compelling, nor is prohibiting all bumper stickers narrowly drawn to achieve such an interest.

A train conductor brought an action against her railroad employer under a federal statute providing liability for work-related injuries occurring on railroads. The employer denied liability, claiming that the conductor's injuries pre-dated her employment and were outside the scope of the statute. At the close of the evidence at trial, the employer moved for judgment as a matter of law (JMOL), which the court denied. The jury returned a verdict for the conductor. The employer has renewed its JMOL motion. What standard should the court apply in ruling on the motion? A: Whether a preponderance of the evidence supports the verdict. B: Whether the verdict is against the weight of the evidence. C: Whether there is a scintilla of evidence to support the verdict. D: Whether there is legally sufficient evidence to support the verdict.

D is correct. Under Federal Rule of Civil Procedure (FRCP) 50, a court may grant judgment as a matter of law against a party only if the court finds that a reasonable jury would not have a legally sufficient evidentiary basis to find for that party.


Conjuntos de estudio relacionados

Cellular Respiration & Photosynthesis

View Set